+ All Categories
Home > Documents > Emergency Medicine Text Review - BluWiki - 2017...

Emergency Medicine Text Review - BluWiki - 2017...

Date post: 25-Mar-2018
Category:
Upload: voxuyen
View: 219 times
Download: 4 times
Share this document with a friend
72
100.A 30-year-old was involved in a rapid deceleration motor vehicle crash at street speed. He was gripping the steering wheel tightly, and complains of pain at the MCP joint of the right thumb. On exam there is tenderness and mild swelling of the joint on the ulnar side. X-rays are negative. Which of the following is correct regarding the evaluation and treatment of this injury? a. If the ulnar collateral ligament demonstrates more than 35 o of laxity when compared to the uninjured side, place a thumb spica splint and refer to orthopedics for probable surgical management. b. If the radial collateral ligament demonstrates any laxity, refer to orthopedics for surgical repair. c. All gamekeepers’ thumbs need to be repaired surgically. d. The absence of a fracture indicates that no splinting will be necessary. e. If the ulnar collateral ligament demonstrates less than 20 o laxity, apply a universal wrist splint, and refer for surgical repair. 101.This x-ray shows a: a. dorsal dislocation of the lunate. b. volar dislocation of the lunate. c. dorsal perilunate dislocation. d. volar perilunate dislocation. e. scapholunate dissociation. 102.Elbow dislocations are associated most commonly with concomitant injuries to the: a. ulnar nerve and radial artery. b. ulnar nerve and brachial artery. c. radial nerve and ulnar artery. d. radial nerve and brachial artery. e. median nerve and radial artery. 103.This child fell while rollerblading and has a tender elbow. This x-ray (figure 169- 15) is most consistent with: a. nursemaid’s elbow. b. radial head fracture. c. proximal ulnar fracture. 37 Rosen 5 th , Figure 44-11 Rosen’s 5 th , Figure 169-15
Transcript
Page 1: Emergency Medicine Text Review - BluWiki - 2017 Bluwikibluwiki.com/images/5/50/Questions_100-200.doc · Web viewMetabolic acidosis without an anion gap is caused by: cyanide poisoning.

100. A 30-year-old was involved in a rapid deceleration motor vehicle crash at street speed. He was gripping the steering wheel tightly, and complains of pain at the MCP joint of the right thumb. On exam there is tenderness and mild swelling of the joint on the ulnar side. X-rays are negative. Which of the following is correct regarding the evaluation and treatment of this injury?

a. If the ulnar collateral ligament demonstrates more than 35o of laxity when compared to the uninjured side, place a thumb spica splint and refer to orthopedics for probable surgical management.

b. If the radial collateral ligament demonstrates any laxity, refer to orthopedics for surgical repair.c. All gamekeepers’ thumbs need to be repaired surgically.d. The absence of a fracture indicates that no splinting will be necessary.e. If the ulnar collateral ligament demonstrates less than 20o laxity, apply a universal wrist splint,

and refer for surgical repair.

101. This x-ray shows a:a. dorsal dislocation of the lunate. b. volar dislocation of the lunate.c. dorsal perilunate dislocation.d. volar perilunate dislocation.e. scapholunate dissociation.

102. Elbow dislocations are associated most commonly with concomitant injuries to the:a. ulnar nerve and radial artery.b. ulnar nerve and brachial artery.c. radial nerve and ulnar artery.d. radial nerve and brachial artery.e. median nerve and radial artery.

103. This child fell while rollerblading and has a tender elbow. This x-ray (figure 169-15) is most consistent with:

a. nursemaid’s elbow.b. radial head fracture. c. proximal ulnar fracture. d. elbow dislocation.e. supracondylar fracture

104. Monteggia’s injury is a :a. fracture of the ulnar shaft with radial head dislocation.b. fracture of the ulnar shaft with distal radioulnar dislocation.c. fracture of both the radius and ulna.d. distal radial fracture with radioulnar dislocation.e. distal radial fracture with radial head dislocation

105. This x-ray (Figure 44-24) demonstrates a: a. Colles fracture.b. Monteggia fracture. c. Galleazzi fracture. d. Bennett fracture.e. Barton fracture.

37

Rosen 5th, Figure 44-11

Rosen’s 5th, Figure 169-15

Rosen 5th, Figure 44-24

Page 2: Emergency Medicine Text Review - BluWiki - 2017 Bluwikibluwiki.com/images/5/50/Questions_100-200.doc · Web viewMetabolic acidosis without an anion gap is caused by: cyanide poisoning.

100. aRosen 5th, Chapter 43, p. 520

Injury to the ulnar collateral ligament (UCL) was first described as an occupational hazard of Scottish gamekeepers who damaged their thumbs by repeatedly twisting the necks of hares. Skiing is now the most common cause of both acute and chronic injury to the UCL. UCL rupture occurs ten times more often than radial collateral ligament injury. The mechanism of injury is forced radial deviation (abduction), and the subsequent tear usually occurs at the insertion into the proximal phalanx. Complete and partial ruptures can usually be differentiated by clinical examination. Valgus stress testing of the ulnar collateral ligaments is required and should be performed in full extension and in 30% flexion to avoid the stabilizing effect of the volar plate. More than 35% of joint laxity or 15% more laxity than is present in the uninjured thumb is consistent with complete UCL rupture. Acute partial ruptures of the UCL can be effectively treated by a 4-week period of immobilization in a thumb spica cast, and full recovery is the rule. Complete ligament tear requires surgical repair because a high number have associated soft tissue interposition from the adductor aponeurosis (Stener’s lesion) with limited predicted healing potential.

101. cRosen 5th, Chapter 44, p. 541

A perilunate dislocation is best seen on the lateral view of the wrist. Although the lunate remains in position relative to the distal radius, the capitate is dorsally dislocated. The PA view shows overlap of the distal and proximal carpal rows and may also demonstrate an associated scaphoid fracture or subluxation.

102. bRosen 5th, Chapter 45, p. 572

In patients with elbow dislocations, neurovascular complications occur in 8 to 21%, the most frequent being injury to the ulnar nerve. Vascular complications occur in 5 to 13% of elbow dislocations, with brachial artery injury the most common.

103. eRosen 5th, Chapter 45, p. 564

Supracondylar injuries are extra-articular fractures which occur most commonly in children; 95% are displaced posteriorly as a result of an extension force. The patient will have significant swelling and tenderness at the elbow. Radiographs may reveal a fat-pad sign in undisplaced fractures due to visualization of fat from the olecranon fossa (posterior fat pad) as it is displaced by the hemarthrosis. In some undisplaced fractures, the fracture line may not be seen, with the fat-pad sign being the only evidence of injury. Treatment should be initiated as though a fracture were identified, with splint immobilization and orthopedic consultation.

104. aRosen 5th, Chapter 44, p. 551

Fracture of the ulnar shaft with a radial head dislocation is often referred to as Monteggia fracture-dislocation. It is typically a diaphyseal fracture in the proximal third of the ulna with an anterior dislocation of the radial head (60% of cases). Anterolateral and posterolateral dislocation of the radial head or a metaphyseal ulna fracture are other possibilities. As a rule, the radial head normally points to the capitellum in all radiographic views of the elbow. In a Monteggia fracture, the apex of the ulna fracture points in the direction of the radial head dislocation.

105. cRosen 5th, Chapter 44, p. 552

A Galeazzi fracture involves the junction of the middle and distal thirds of the radius, with an associated dislocation or subluxation of the distal radial-ulnar joint (DRUJ). This rare injury accounts for only 7% of all fractures of the forearm. It occurs as a result of a fall on the outstretched hand, with the wrist in extension and the forearm forcibly pronated. The radial fracture causes obvious swelling and deformity on the radial side of the forearm, and pain is localized over the fracture site. In addition, the DRUJ is swollen and painful on palpation, and the ulnar head appears prominent when compared with the unaffected wrist.

38

Page 3: Emergency Medicine Text Review - BluWiki - 2017 Bluwikibluwiki.com/images/5/50/Questions_100-200.doc · Web viewMetabolic acidosis without an anion gap is caused by: cyanide poisoning.

106. A 16 year old boy complains of pain and swelling in his right hand after hitting a wall with a closed fist. You know that:

a. the neck of the metacarpal is the most frequently fractured segment.b. metacarpal fractures of the long and index finger do not need to be reduced if there is less than

30o of angulation.c. metacarpal fractures of the ring and small fingers do not need to be reduced if there is less than

20o of angulation.d. most metacarpal fractures angle in a volar direction.e. a boxer’s fracture refers to metacarpal neck fractures of the long and sometimes ring fingers.

107. A 5 year old boy falls and fractures his radius. a. Salter-Harris Type I fractures are the most common type b. The picture shows a Salter-Harris Type II fracture. c. Salter-Harris Type III fractures involve the epiphyseal plate and

have a widened space between epiphysis and metaphysis.d. Salter-Harris Type IV fractures are difficult to diagnose using

standard radiographs.e. Salter-Harris Type I, II, and III injuries have poor prognosis despite

proper reduction and immobilization.

108. A 29 year-old man is tackled while playing football and suffers what appears to be a shoulder dislocation. You know that:

a. the most common direction of dislocation is posterior.b. anterior-posterior (AP) and lateral radiographs are the best views to confirm the diagnosis.c. after reduction he should be placed in a shoulder immobilizer for 2 months.d. infraglenoid dislocations are associated with seizures.e. inadequate muscle relaxation is the most common reason for failed reduction.

109. A 43-year-old softball player pulled up lame stretching a single into a double. You suspect a ruptured Achilles tendon, since you know:

a. when the soleus and gastrocnemius muscles contract, the Achilles tendon pulls up the calcaneus, dorsiflexing the foot.

b. rupture often occurs in professional sports settings, especially in over-conditioned athletes.c. the patient hears a popping sound and then has difficulty walking.d. the Thompson-Doherty test is performed by squeezing the midportion of the calf while the patient

is lying in the prone position; an intact Achilles tendon is demonstrated by dorsiflexion of the foot.

e. unbearable pain is a cardinal symptom.

110. A 23-year-old hairdresser complains of two days of increasing pain and swelling in her right index finger pad. She has no fever. Her finger pad is tense, red, warm, and very tender. Her nail and paronychial region are not affected. You should:

a. place her on amoxicillin / clavulanate (Augmentin®), elevation, and warm soaks.b. perform a through-and-through incision of the pad volar to the neurovascular bundle, then

amoxicillin / clavulanate (Augmentin®), elevation, and warm soaks.c. perform a fish mouth incision and refer to a hand surgeon.d. perform a unilateral longitudinal incision on the ulnar side dorsal to the neurovascular bundle,

amoxicillin / clavulanate (Augmentin®), elevation, warm soaks, pain medicine, recheck in two days.

e. remove the nail, incise the eponychium, hot soaks, pain meds, and recheck in two days.

39

Page 4: Emergency Medicine Text Review - BluWiki - 2017 Bluwikibluwiki.com/images/5/50/Questions_100-200.doc · Web viewMetabolic acidosis without an anion gap is caused by: cyanide poisoning.

106. dRosen 5th, Chapter 43, p. 513

Fractures of the metacarpal neck are among the most common fractures in the hand. They are usually due to a direct impaction force. A fracture of the fifth metacarpal neck is often referred to as a boxer’s fracture. These fractures are usually unstable with volar angulation. Angulation of less than 20° in the fourth and 40° in the fifth metacarpal will not result in functional impairment. In the second and third metacarpal, angulation of <15° is acceptable. These fractures should be splinted with the wrist in 20° extension and the MP flexed at 90°. Fractures of the second or third metacarpal that are significantly displaced or angulated require anatomic reduction and surgical fixation.

107. cRosen 5th, Chapter 169, p. 2373

108. eRosen 5th, Chapter 46, p. 592-594

Dislocation of the glenohumeral joint is the most common major joint dislocation. Anterior dislocations are by far the most common. Although the anteroposterior radiograph will reveal the dislocation, the scapular Y radiograph will indicate the direction of dislocation: anterior or posterior. Since severe pain and muscle spasms are the norm, muscle relaxation and analgesia are paramount. Posterior glenohumeral dislocations may occur during a fall or from violent muscle contraction from a seizure or electric shock. In uncomplicated cases the shoulder is immobilized for 3 to 6 weeks in younger patients and 1 to 2 weeks in older (over age 40 years) individuals.

109. cRosen 5th, Chapter 51 p.716

Achilles tendon rupture usually occurs in middle-age persons during sporadic or intermittent involvement in recreational sports. Injury mechanism is sudden unexpected dorsiflexion, forced dorsiflexion of a plantarflexed foot, and strong push-off of the foot with simultaneous knee extension and calf contraction (e.g., a runner accelerating from the starting position). The diagnosis is primarily clinical. Patients usually describe a sudden onset of pain at the back of the ankle associated with an audible “pop” or “snap.” Although the pain may resolve rapidly, weakness in plantarflexion persists. The classic maneuver to assess the integrity of the Achilles tendon is the Thompson test. This is performed with the patient prone and the knee flexed at 90o. Alternatively, the patient may kneel on a chair with both knees flexed at 90o and the feet dangling over the edge. Squeezing the calf muscles in these two positions should cause passive plantarflexion of the foot. Absence of this motion or a weakened response compared with the uninjured side suggests complete rupture.

110. dRosen 5th, Chapter 43, p. 529-530

A felon is a subcutaneous pyogenic infection of the pulp space of the distal finger or thumb. The septa of the finger pad produce multiple individual compartments and confine the infection under pressure. The patient presents with throbbing pain and a red, tense distal pulp space. Staphylococcus aureus is the most common organism. If the finger pad is swollen and tense, or if there is any palpable fluctuance, drainage must be undertaken. A unilateral longitudinal approach spares the sensate volar pad and achieves adequate drainage. Introduce a #11 blade lateral to the paronychium and direct it in a volar direction until you find pus. Extend the incision ensure adequate drainage, but not to the distal interphalangeal (DIP) flexor crease.

40

I II III IV VRosen 5th, Table 169-1

Page 5: Emergency Medicine Text Review - BluWiki - 2017 Bluwikibluwiki.com/images/5/50/Questions_100-200.doc · Web viewMetabolic acidosis without an anion gap is caused by: cyanide poisoning.

111. Which of the following is true of this injury?a. This is a common wrist fracture.b. The lunate requires immediate reduction.c. The distal radial-ulnar ligament is disrupted and will

require surgical repair.d. The clenched fist AP view may accentuate this finding.e. None of the above – this is a normal x-ray.

112. A 38-year-old man developed severe lower back pain that shoots down his right leg as he was lifting a large box at work. He has no previous history of back problems. He has no bowel or bladder symptoms, and denies genital numbness. On exam he has no back deformity or palpable pain, but when he raises his leg off of the bed, a severe pain shoots down his leg to his great toe. He has some numbness of the lower leg and web space next to the great toe. He has 4/5 strength with dorsiflexion of the foot. His deep tendon reflexes are normal. He most likely has:

a. herniated nucleus pulposus (HNP) of L1-L2.b. HNP of L3-L4.c. HNP of L5-S1.d. severe lumbar strain.e. drug seeking behavior, since the findings are non-anatomic.

113. A 34-year-old man is ejected from a car. He has no head injury, but complains of severe pain in his pelvis. His airway and breathing are fine, but he is hypotensive – BP 80/40 mm Hg, heart rate 120. His workup shows multiple abrasions and pelvic pain. His pelvis x-ray is shown. His abdominal CT shows no intra-abdominal injury. After 4 liters of IV fluid, his blood pressure is 95/60. Of the choices given below, your most appropriate would be to:

a. re-examine by compression and distraction of pelvic wings.

b. transfuse fresh frozen plasma to control bleeding.c. obtain stat vascular consult for intraoperative exploration and cauterization of vessels.d. apply pneumatic antishock garment (PASG) to act as splint and compressive dressing.e. arrange for hanging traction with at least 20 pounds of weights.

114. A 22-year-old man was in a head-on motor vehicle crash. His right leg is in adduction, flexion, and internal rotation. Which is the correct statement about this injury?

a. This is likely a femoral neck fracture, so x-rays and pain relief should be given. b. After sufficient analgesia and sedation, gentle flexion to 90o with upward traction should be

performed as soon as possible. c. The most likely associated injury is to the femoral nerve. d. The risk of associated acetabular fracture is minimal. e. The risk of avascular necrosis is >50%.

115. Ankle sprains:a. are common in young children and toddlers.b. most commonly involve the anterior talofibular ligament.c. most commonly occur on the medial side of the ankle.d. require radiography to diagnosis a third-degree injury.e. involving isolated deltoid ligament injuries are very common.

41

Rosen 5th, Figure 44-10

Rosen 5th, Figure 48-6

Page 6: Emergency Medicine Text Review - BluWiki - 2017 Bluwikibluwiki.com/images/5/50/Questions_100-200.doc · Web viewMetabolic acidosis without an anion gap is caused by: cyanide poisoning.

111. eRosen 5th, Chapter 44, p. 538

A stage I injury, or scapholunate dissociation, results in a characteristic widening of the scapholunate joint on the PA view, which has been called the Terry-Thomas sign, after the British comedian with a gap between his front teeth. When a scapholunate ligament injury is suspected clinically, additional stress views should be obtained. Views taken with a clenched fist and ulnar deviation (the clenched fist AP view) will accentuate widening of the scapholunate joint.

112. cRosen 5th, Chapter 47, p. 617

The most important test for back pain is the straight leg raise. This is done with the patient supine and the legs extended, or with both legs slightly flexed at the hips and knees. The symptomatic leg is passively raised, with the knee fully extended. Pain caused by a disk herniation in L4-L5-S1 usually results in increased pain in the leg at anywhere from 30 to 70o of elevation, as the affected nerve root is stretched. It is important to note that back pain, probably secondary to stretching of the paravertebral muscles as the leg is elevated, indicates absolutely nothing about radiculopathy. A straight leg raise test is positive only if lifting the affected leg reproduces or worsens the pain in that leg. The pain of radiculopathy is usually worse in the leg than in the back and almost always radiates past the knee. Ninety-five percent of disk herniations occur at L4–L5 or L5–S1. Without complaint of sciatica, or pain below the knee in a dermatomal distribution, the chance of a herniated nucleus pulposus is 1 in 1000.

113. dRosen 5th, Chapter 48, p. 640

When attempts to stabilize the patient hemodynamically with crystalloid infusion and transfusion of blood products are unsuccessful, application of a pneumatic anti-shock garment should be discussed with orthopedic and general surgical consultants. Although controversial, PASG can temporarily splint the fracture and tamponade pelvic bleeding until definitive stabilization and angiography with embolization can be performed..

114. bRosen 5th, Chapter 49, pages listed in answer

The Allis maneuver (Figure 49-27) should be attempted in order to reduce a [posterior hip dislocation as quickly as possible. The patient is placed supine the knee is flexed and steady traction is applied in line with the deformity – the hip is brought slowly to 90o of flexion after reduction, the hip is extended while maintaining traction (p. 664). A patient with a femoral neck fracture presents with leg externally rotated and shortened. A patient with a posterior hip dislocation typically holds the hip flexed, adducted, and internally rotated (p. 662). Sciatic palsies, especially the peroneal nerve branch, occur in 10% of patients with posterior hip dislocation (p. 662). Acetabular fractures are common (p. 662). Avascular necrosis is reported to occur in 1% to 17% of patients with this injury (p. 665).

115. bRosen 5th, Chapter 51 p.714

Most ankle sprains occur from extreme inversion and plantarflexion. Approximately two thirds of ankle sprains are isolated anterior talofibular ligament injuries, whereas 20% involve both anterior talofibular and calcaneofibular ligament injuries. Isolated injury of the deltoid ligament occurs in fewer than 5% of ankle sprains. A grade III sprain involves a complete tear of the ligament with marked joint instability and severe edema and ecchymosis, and is a clinical diagnosis.

42

Page 7: Emergency Medicine Text Review - BluWiki - 2017 Bluwikibluwiki.com/images/5/50/Questions_100-200.doc · Web viewMetabolic acidosis without an anion gap is caused by: cyanide poisoning.

116. A 75-year-old woman fell at a local flower shop and complains of severe left hip pain. Her left leg is shortened and externally rotated. She has strong pedal pulses and sensation is intact. Thorough head-to-toe examination reveals no other injuries. Her left hip x-ray is shown (Figure 49-17), so you arrange for the patient’s admission to the orthopedic doctor on call. The patient’s daughter, a pediatrician, wants to know about potential complications from this injury. You tell her that, just as in children, the most common complication is:

a. avascular necrosis. b. hemorrhage.c. osteomyelitis.d. fat embolism.e. femoral nerve injury.

117. A 24 year-old basketball player severely twisted his left knee several hours ago and is unable to bear weight. There is a large joint effusion. While examining his knee, you see that valgus stress at 30o flexion causes more than a centimeter of “give” without a firm endpoint. The knee is stable in extension with valgus stress. This suggests:

a. an isolated medial collateral ligament disruption.b. an isolated lateral collateral ligament disruption.c. a medial collateral ligament tear with probable involvement of the cruciates and posterior capsule. d. a lateral collateral ligament rupture with posterior cruciate tear. e. probable anterior cruciate rupture with meniscal tear.

118. A 32-year-old man swung at a pitch while playing softball, then collapsed due to severe knee pain. His injured leg looks like this. Your treatment should be:

a. immobilization, x-ray, pain medicine, and examination for probable peroneal nerve injury.

b. needle arthrocentesis of the prepatellar bursa, then ice and elevation.c. orthopedic consult for repair of the patellar tendon.d. gentle lateral pressure on the patella with the knee in extension.e. examine for possible popliteal artery disruption.

119. A 35-year-old male skier fell and twisted his knee. Within five minutes, he developed severe pain and a large joint effusion. When testing his knee stability, you know that:

a. the anterior drawer sign is highly accurate in diagnosing acute ACL injuries.b. abduction, flexion, and internal rotation of the femur on the tibia produce injuries to the lateral

side of the knee.c. adduction, flexion, and external rotation of the femur on the tibia produce injuries to the medial

side of the knee.d. lateral knee injuries are far more common than medial knee injuries.e. a negative Lachman’s test rules out ACL injury with >90% accuracy.

120. Metabolic acidosis without an anion gap is caused by:a. cyanide poisoning.b. renal tubular acidosis.c. lactic acidosis.d. ketoacidosis.e. salicylate ingestion.

43

Rosen 5th, Figure 49-17

Page 8: Emergency Medicine Text Review - BluWiki - 2017 Bluwikibluwiki.com/images/5/50/Questions_100-200.doc · Web viewMetabolic acidosis without an anion gap is caused by: cyanide poisoning.

116. aRosen 5th, Chapter 49, p. 656

In a patient with femoral neck fracture, avascular necrosis is the most common complication, despite optimal treatment, because of the complex arterial anatomy. The mortality rate during the first year after a femoral neck fracture is 14%, compared with 9% for the control population. Deep infection, in the form of osteomyelitis or septic arthritis, is more common with femoral neck fractures because the fracture line extends into the joint. Pulmonary embolism is the leading cause of death at 7 days post-fracture in all orthopedic patients.

117. aRosen 5th, Chapter 50, p. 678

Abduction or valgus pressure applied to a knee in approximately 30° of flexion determines the integrity of the medial capsular and ligamentous structures. If there is a demonstrated laxity of greater than 1 cm without a firm end-point as compared to the other knee, there is a complete rupture of the medial collateral ligament. If there is laxity with a firm end-point or a laxity of less than 1 cm, an incomplete or partial tear is present. If there is no demonstrated instability but there is pain, the patient has suffered a strain. The patient who is unstable with the valgus test performed with 30° of flexion should be brought into full extension, if possible, and similar maneuvers carried out. Medial instability in full extension indicates a severe lesion involving the cruciate ligaments and posterior capsule along with the medial ligaments.

118. dRosen 5th, Chapter 50 p.688

Patellar dislocation may occur from an athletic injury caused by valgus stress combined with flexion and external rotation. Patients may complain of the knee giving out accompanied by pain and swelling. Inability to bear weight or to flex the knee is a common complaint. Examination shows an anterior defect with the patella deviated laterally. Closed reduction is accomplished through force or pressure directed anteromedially on the lateral patellar margin while simultaneously gently extending the leg.

119. eRosen 5th, Chapter 50 p.677

A tear in the anterior cruciate ligament or intra-articular fracture usually results in an immediate hemarthrosis and inability to continue activity. A torn meniscus may cause an acutely locked knee but more commonly has delayed onset of swelling over 12 to 24 hours and intermittent locking associated with joint line pain. Lachman’s test is currently the single best clinical test for determining the integrity of the ACL and one of the only reliably performed tests in a patient with an acute hemarthrosis. Accuracy in diagnosing ACL injury increases from 70% to 99% using Lachman’s test rather than the anterior drawer test. The anterior drawer test is not reliable and is of little value in diagnosing acute ACL injuries.

120. bRosen 5th, Chapter 118 p.1719

Elevated Anion Gap (“CAT MUD PILES”)Carbon monoxide / Cyanide exposureAcute alcohol intoxication / Alcoholic ketoacidosisToluene exposureMethanol intoxicationUremiaDiabetic ketoacidosisParaldehyde ingestionIsoniazid (INH) / Iron intoxicationLactic acidosisEthylene glycol intoxicationSalicylate intoxication

Normal Anion GapGastrointestinal loss of HCO3

Diarrhea Enterostomy UreteroenterostomyRenal loss of HCO3

Renal tubular acidosis Acetazolamide Hyperalimentation

44

Page 9: Emergency Medicine Text Review - BluWiki - 2017 Bluwikibluwiki.com/images/5/50/Questions_100-200.doc · Web viewMetabolic acidosis without an anion gap is caused by: cyanide poisoning.

121. Which statement is correct concerning the entity shown in this x-ray?a. This is a common foot injury, usually caused by inversion. b. Associated fractures are quite common, and usually involve the base

of the 2nd metatarsal.c. This is a pseudo-pseudo-Jones fracture. d. Rest, ice, elevation, and ambulation as tolerated give the best long-

term results.e. You can easily reduce this injury with gentle axial traction and plantar

flexion.

122. A 35-year-old man accidentally chopped off his left index finger at the PIP joint about six hours ago while cutting firewood at a campsite in a local state park. He stuck the amputated finger in his pocket, hiked out of the woods, and drove to your Emergency Department. You should:

a. clean the amputated finger with Betadine® and place it in the refrigerator.b. clean the amputated finger with hydrogen peroxide, wrapped in saline gauze, and place it on ice.c. irrigate the amputated finger with saline, wrap it in sterile dry gauze and place it in a plastic bag,

which should be immersed in ice.d. Irrigate the amputated finger with saline, wrap it in saline-moistened gauze and place it in a

plastic bag, which should be immersed in ice-water slurry.e. There is no need to care for the amputated extremity as the ischemic time is too long and no

reimplantation will be possible

123. A 20-year-old college student has redness, tenderness, and swelling on the plantar surface of his foot. Two days ago, while wearing sneakers, he stepped on a nail. You should prescribe:

a. cephalexin.b. amoxicillin/clavulanate.c. ciprofloxacin.d. tetracycline.e. erythromycin.

124. Antibiotics are clearly indicated for:a. all dog bites.b. all cat bites.c. all human bites.d. all foot puncture wounds.e. all intraoral lacerations.

125. The acute life threats in adrenal inufficiency are:a. hypertension and hyperglycemia.b. hyperthermia and hypoglycemia.c. hypothermia and hyperglycemia.d. hypotension and hypoglycemia.e. hyperthermia and hypertension.

126. A patient being treated for diabetic ketoacidosis develops weakness and paresthesias. The most likely metabolic abnormality is:

a. acidosis.b. hypokalemia.c. hypophosphatemia.d. hypocalcemia.e. hypoglycemia.

45

Rosen 5th, Figure 51-24

Page 10: Emergency Medicine Text Review - BluWiki - 2017 Bluwikibluwiki.com/images/5/50/Questions_100-200.doc · Web viewMetabolic acidosis without an anion gap is caused by: cyanide poisoning.

121. bRosen 5th, Chapter 51, p. 726

The six-bone tarsometatarsal complex is known as the Lisfranc joint. Up to 20% of fractures at this joint are missed in the ED. The force required and the mechanisms of injury are varied and can range from a seemingly minor rotational force to severe axial load as seen in an automobile accident. The great majority of injuries to the Lisfranc joint are associated with fractures, usually of the metatarsals, the cuboid, or the cuneiforms. A fracture of the base of the second metatarsal is pathognomonic of a disruption of the ligamentous complex. Injuries to the Lisfranc joint frequently require open reduction and fixation or percutaneous placement of Kirschner wires and non-weight-bearing for several weeks.

122. dRosen 5th, Chapter 43, p. 527

The time that an amputated part can survive before reimplantation has not been determined. As a general rule, the more proximal the amputation, the less ischemic time the amputated part can tolerate. Warm ischemia may be tolerated for 6 to 8 hours, but there are reports of successful replantation of digits after 30 hours of warm ischemia. The amputated part requires minimal handling and should be cooled as soon as possible. After wrapping in saline-moistened gauze, the part is sealed in a dry plastic bag and placed in ice water. Ice should not come in direct contact with the tissue because this can cause local damage. Local antiseptics, especially hydrogen peroxide or alcohol, should not be used because they may damage viable tissues.

123. cRosen 5th, Chapter 52, p. 748

The infection rate for foot puncture wounds has been reported to be as high as 15%. Most infections are due to gram-positive organisms, with Staphylococcus aureus predominating, followed by other staphylococcal and streptococcal species. Pseudomonas aeruginosa is the most frequent pathogen isolated post-puncture wound osteomyelitis, particularly when foreign-body penetration occurs through the sole of an athletic shoe. Because this organism is not detected in new shoes, it has been postulated that the foam rubber material becomes colonized in the warm, humid summer months. The only oral agents consistently effective against Pseudomonas are the fluoroquinolones, including ciprofloxacin and levofloxacin.

124. bRosen 5th, Chapter 52, p. 748

Antibiotic prophylaxis is required for all patients with cat bites. Antibiotic prophylaxis for dog bites is more controversial, and seven of eight randomized trails of dog bite wounds show no benefits with antibiotics. While human bites of the hand are at a high risk for infection, human bites of other locations appear to have no worse infection rate than other wounds. No data suggest a benefit from prophylactic antibiotics in foot puncture wounds. Little data suggest a clear indication for prophylactic antibiotics in patients with intraoral lacerations, but through-and-through wounds have an infection rate twice as high as simple mucosal injuries.

125. eRosen 5th, Chapter 122, p. 1781

The acute life threats in adrenal insufficiency are hypotension and hypoglycemia. Hypotension responds well to glucocorticoid replacement with IV hydration, and hypoglycemia to IV administration of D5W.

126. bRosen 5th, Chapter 120, p. 1753-1754

Potassium replacement is invariably needed in DKA. The initial potassium level is often normal or high despite a large deficit because of severe acidosis. Potassium levels often plummet with correction of acidosis and administration of insulin. Potassium should be administered with the fluids while the laboratory value is in the upper half of the normal range.

46

Page 11: Emergency Medicine Text Review - BluWiki - 2017 Bluwikibluwiki.com/images/5/50/Questions_100-200.doc · Web viewMetabolic acidosis without an anion gap is caused by: cyanide poisoning.

127. According to his father, this 11-year-old boy has complained about a vague hip, thigh, and knee pain for the past 2 weeks. He recalls no injury. There has been no fever. The child walks with a limp and winces when his right foot hits the ground. His x-ray demonstrates:

a. osteomalacia femoralis.b. Osgood-Schlatter disease.c. Legg-Calvé-Perthes disease.d. septic synovitis.e. slipped capital femoral epiphysis.

128. In a patient with primary adrenal insufficiency you would expect to see:a. hypernatremia.b. hypokalemia.c. hypoglycemia.d. azotemia.e. acidosis.

129. The most common cause of a markedly elevated serum potassium is:a. laboratory error.b. renal failure.c. Addisonian crisis.d. rhabdomyolysis.e. acidosis.

130. Hirschsprung's disease:a. is typically diagnosed in infancy.b. almost always is associated with enterocolitis.c. is more frequent in females.d. can be diagnosed by ultrasound.e. can best be managed pharmacologically.

131. You deliver a child at 36-weeks gestation and intestines are protruding from a defect in the abdominal wall. On inspection, you note that the intestines lack a peritoneal covering.

a. A gastroschisis is a defect in the umbilical ring that allows the intestines to protrude out of the abdominal cavity in a sac.

b. An omphalocele is a defect in the abdominal wall that allows the evisceration of abdominal structures without a sac being present.

c. Gastroschisis and omphalocele are invariably fatal within hours.d. You are twice as likely to encounter gastroschisis you are to see omphalocele.e. Emergency department treatment gastroschisis involves gastric decompression; this same

treatment is contraindicated in omphalocele.

132. A 13-year-old male complains of acute, severe pain and swelling in his left testicle. He says that the pain woke him from sleep. He had something silimar a week or two ago, but it got better spontaneously. Suspecting torsion testicle, you know that:

a. absence of the cremasteric reflex is not a useful physical exam finding. b. if this patient's symptoms have only been present for 1 hour, Doppler imaging of the testicles can

be obtained before obtaining surgical consultation. c. relief of this pain with elevation of the scrotum indicates that epididymitis is the diagnosis. d. torsion of the right testicle is more common due to a longer spermatic cord.e. testicular pain may be constant or intermittent, but it is not positional.

47

Rosen 5th, Figure 49-34

Page 12: Emergency Medicine Text Review - BluWiki - 2017 Bluwikibluwiki.com/images/5/50/Questions_100-200.doc · Web viewMetabolic acidosis without an anion gap is caused by: cyanide poisoning.

127. eRosen 5th, Chapter 169 p.2387

Children with a stable slipped capital femoral epiphysis have symptoms of intermittent limp and pain for several weeks to months. Stable slips make up approximately 90% of all cases. The pain may be localized to the hip but more commonly is poorly localized to the thigh, groin, or knee. On AP radiographs, signs of slippage include Klein’s line drawn along the superior margin of the femoral neck. With a normal hip, the line intersects or falls within the epiphysis, whereas in a hip with a slipped epiphysis, the line does not come in contact with the epiphysis.

128. cRosen 5th, Chapter 122 p.1781

Two thirds of patients with adrenal failure have associated hypoglycemia. The symptoms are characteristic of hypoglycemia: perspiration, tachycardia, weakness, nausea, vomiting, headache, convulsions, and coma. The pathophysiology is decreased gluconeogenesis and increased peripheral glucose use secondary to lipolysis. Electrolyte abnormalities are common. Hyponatremia is present in 88% of cases, hyperkalemia in 64%, either hyponatremia or hyperkalemia in 92%, and hypercalcemia in 6% to 33%.

129. aRosen 5th, Chapter 119, p. 1730

When faced with a report of a high serum potassium level, the Emergency Physician should first consider the possibility of laboratory error, the most common cause of hyperkalemia. Hemolysis during phlebotomy, as may occur when blood is obtained with a small needle or sampled in a high-vacuum tube, releases potassium into the sample and causes a spuriously high potassium level to be measured.

130. aRosen 5th, Chapter 165, p. 2305-2306

Hirschsprung’s disease accounts for 20% of partial intestinal obstruction early in infancy. It is 4 to 5 times more common in boys and can be associated with Down syndrome or a variety of other anomalies of the GI, GU, or neurologic systems. Neonates often present in the nursery with a failure to pass meconium. Diagnosis is suspected on barium enema, and confirmed by biopsy. Treatment in surgical.

131. dRosen 5th, Chapter 9, p. 103

Gastroschisis is a defect in the abdominal wall that allows the antenatal evisceration of abdominal structures without a sac being present. Omphalocele is a defect in the umbilical ring that allows the intestines to protrude out of the abdominal cavity in a sac. Gastroschisis occurs twice as frequently as omphalocele. ED treatment involved gastric decompression and placing an occlusive plastic covering to prevent heat and water loss.

132. eRosen 5th, Chapter 94 p.1423

Manual detorsion is never curative but should be attempted in most cases while awaiting surgical treatment. Prehn's sign, or relief of pain with elevation of the scrotum, was historically thought to be associated with epididymitis, but this association is false and should not be used to distinguish torsion from epididymitis. The cremasteric reflex is almost always absent in testicular torsion. The salvage rate in testicular torsion remains high if pain has lasted less than 6 hours, but a urology consult should be obtained as soon as the diagnosis seems likely clinically.

48

Page 13: Emergency Medicine Text Review - BluWiki - 2017 Bluwikibluwiki.com/images/5/50/Questions_100-200.doc · Web viewMetabolic acidosis without an anion gap is caused by: cyanide poisoning.

133. Concerning intraosseous lines:a. Fat embolism is a common complication, especially in adult patients.b. In adults, the tibia is technically easier than in children because of relative proximity to the

surface. c. The preferred insertion site in the adult femur is the lateral surface of the trochanter.d. Infection rates approach 20% in both adults and children. e. Any drug which can be given intravenously can also be delivered through an intra-osseous line.

134. A 3-month-old girl is irritable, feeds poorly, and breaks into a profuse sweat whenever she takes the nipple. She is tachypneic and pale. Her heart rate is 280 / minute. Chest x-ray shows a large heart with alveolar infiltrates. A rhythm strip is shown. You know that:

a. jugular venous distension and peripheral edema are commonly found in infants with this condition.

b. volume replacement is the most essential treatmentc. heart rate is the least malleable of the cardiovascular parameters.d. cardioversion with 0.01 watt-sec/kg is indicated for profound shocke. a stable patient can be treated with vagal maneuvers

135. A 16-month-old child is brought to the emergency department immobilized in cervical spine precautions. The child was an unrestrained passenger in a moderate-speed rapid-deceleration motor vehicle collision. You are concerned about possible neck injury, keeping in mind that:

a. pseudosubluxation of C3 on C4 is common in children. b. the small neck muscles make fractures more common than ligamentous injury. c. the predental space should not exceed 4 to 5 mm in children younger than 10. d. anatomic features of the cervical spine approach adult patterns at around 12 years of age.e. if the child was ambulatory at the scene, spinal precautions are unnecessary.

136. A 2-year-old child is brought to the emergency department because of fever and pulling at his left ear. Otitis media is best confirmed by:

a. decreased mobility of his tympanic membrane and loss of normal landmarks.b. hyperemia of his tympanic membrane in the absence of crying.c. retraction of his tympanic membrane accompanied by upper respiratory infection symptoms.d. loss of the tympanic light reflex.e. the presence of otalgia and upper respiratory infection symptoms.

137. An 18-month-old boy has fever and poor feeding. His ears look normal, but he has an exudative pharyngitis and enlarged cervical lymph nodes. A rapid antigen detection test for Group A beta-hemolytic streptococcus (GABHS) is negative.

a. Rapid streptococcal tests are highly sensitive, so it is highly unlikely that this child has acute GAS pharyngitis.

b. While GABHS is unlikely at this age, detection is important because of the significant risk of acute rheumatic fever in untreated patients.

c. If the test had been positive, appropriate treatment would include a dose of intramuscular procaine penicillin.

d. If the test had been positive, early institution of appropriate antibiotics would likely have shortened the clinical course of the illness

e. Infectious mononucleosis is not a consideration because of the child’s age.

49

Page 14: Emergency Medicine Text Review - BluWiki - 2017 Bluwikibluwiki.com/images/5/50/Questions_100-200.doc · Web viewMetabolic acidosis without an anion gap is caused by: cyanide poisoning.

133. eRosen 5th, Chapter 8, p. 84

For pediatric patients up to 5 years of age, the tibia is the preferred site. In adults, the most commonly used site is the medial malleolus. Although complications such as fat and marrow emboli to the lungs have been reported, recent data suggest that the risk is not increased by the use of IO access for emergent fluid and drug administration. The tibia is technically more difficult in adults than in children because the adult bone is thicker and the needle tends to slip off. The incidence of infection, including both cellulitis and osteomyelitis, is less than 1%, and the potential for infection can be minimized by limiting the duration of intraosseous infusion and avoiding hypertonic solutions.

134. eRosen 5th, Chapter 164 p. 2290-2292

The predominant symptoms of congenital heart disease include poor feeding, excessive diaphoresis, irritability or lethargy with feeding, weak cry and, in severe cases, grunting and nasal flaring. Tachypnea is a cardinal sign. Since feeding is the infant’s primary form of exertion, dyspnea and sweating during feeding can often be elicited in the history. Peripheral edema, jugular venous distention, and rales are unusual and late signs in infants. Heart rate is the most malleable of the cardiac physiologic parameters. The common pediatric dysrhythmia is paroxysmal supraventricular tachycardia. Initial management of unstable patients with narrow complex tachycardia consists of immediate synchronized cardioversion at 0.5 J/kg with increases in power output to 2 J/kg as needed. In the stable patient, vagal maneuvers are the intervention of choice. If vagal maneuvers are not successful in the stable patient, IV adenosine (0.1 mg/kg –maximum first dose 6mg) followed by (0.2mg/kg – maximum 12mg) can be used.

135. cRosen 5th, Chapter 32, p. 274-276

On a lateral cervical spine view the distance between the anterior aspect of the odontoid process and the posterior aspect of the anterior ring of C1, the so-called predental space, should not exceed 5 mm in a child. Pseudosubluxation of C2 on C3 occurs in approximately 40% of children up to adolescence. Anatomic features of the cervical spine approach adult patterns between the ages of 8 and 10 years. Compared to adults, the child has relatively underdeveloped neck musculature and a head that is disproportionately large and heavy compared to the body, leading to fewer fractures and more ligamentous injuries.

136. aRosen 5th, Chapter 67, p. 928-931

The most significant sign of acute otitis media (AOM) is the loss of or decrease in mobility of the tympanic membrane. The light reflex is of no diagnostic value. The normal eardrum is translucent and pearly gray but may become reddened with crying. The TM of AOM is usually opaque, hyperemic, and sometimes bulging, and bony landmarks (long and short process of the malleus) are not easily discernible.

137. dRosen 5th, Chapter 70, p. 969-972

Early antibiotic treatment of streptococcal pharyngitis leads to a 13% earlier resolution of symptoms and shortens the course of illness by about 1 day. GABHS is primarily a disease of children 5 to 15 years old and occurs in winter and early spring. It is responsible for less than 15% of pharyngitis in patients older than 15 years of age and is rare in patients less than 3 years old. Rapid strep tests have reported specificity of 70% to 100% (with most being >95%) and sensitivity of 31% to 100% (with most being 60% to 95%). The incidence of rheumatic fever parallels that of GABHS, with the peak incidence in children 5 to 15 years old, less common in adults, and rare in children less than 3 years of age. Monospot tests are positive in about 30% of children 0 to 20 months with mononucleosis.

50

Page 15: Emergency Medicine Text Review - BluWiki - 2017 Bluwikibluwiki.com/images/5/50/Questions_100-200.doc · Web viewMetabolic acidosis without an anion gap is caused by: cyanide poisoning.

138. A 3-day-old baby presents to the emergency department with purulent conjunctivitis. His mother has a history of syphilis treated prior to this pregnancy. She had no prenatal care, but delivered in the hospital. The infant received topical erythromycin prophylaxis in the nursery. The most likely cause of this child’s illness is:

a. syphilis.b. Neisseria gonorrhoeae.c. group B streptococcus.d. Herpes simplex.e. Chlamydia trachomatis.

139. A 2-year-old patient has screamed inconsolably for four hours and is brought to the ED by his frazzled parents. He is afebrile and non-toxic in appearance; initial exam shows nothing but mild left conjunctival erythema without discharge. Your next step should be:

a. lumbar puncture.b. Schiotz tonometry.c. fluorescein staining of the cornea.d. urinalysis.e. stool for occult blood.

140. Parents bring a 6-week-old infant to the ED because of 3 days of vomiting, occurring immediately after eager feedings. The baby is fussy, but alert. His mucous membranes are tacky. When you feed him, he eagerly takes the bottle, but then promptly vomits. Except for the dehydration, you find nothing else on the exam. This child will require:

a. surgical intervention.b. intravenous hydration and observation.c. prompt parenteral antibiotic therapy.d. discharge with BRAT diet and parental reassurance.e. change to a soybean-based formula.

141. Cyanosis frequently occurs with:a. ventricular septal defect.b. atrial septal defect.c. patent ductus arteriosus.d. tetralogy of Fallot.e. hypertrophic cardiomyopathy.

142. Regarding ear infections:a. The tympanic membrane should always appear normal in acute otitis externa. b. Normal tympanic membrane mobility rules out acute otitis media.c. Children with acute otitis media who are compliant with appropriate antibiotic regimens do not

require any follow-up.d. An intense worsening of the pain usually indicates sudden tympanic membrane perforation.e. Topical antibiotic therapy for otitis externa is ineffective unless the canal is free of debris and

cerumen.

51

Page 16: Emergency Medicine Text Review - BluWiki - 2017 Bluwikibluwiki.com/images/5/50/Questions_100-200.doc · Web viewMetabolic acidosis without an anion gap is caused by: cyanide poisoning.

138. eRosen 5th, Chapter 66, p. 917

Neonatal conjunctivitis (ophthalmia neonatorum) occurs within the first month of life. The chemical irritation from antimicrobial prophylaxis against bacterial infection occurs within 1-2 days of birth. Other causes include Chlamydia trachomatis, H. influenzae, and Streptococcus pneumoniae. Due to mandated use of erythromycin ocular prophylaxis, Neisseria gonorrhoeae is not a major cause of neonatal conjunctivitis in the United States. This drug, however, has not been proven effective against chlamydia infections. Gonococcal conjunctivitis generally has its peak time of onset between 3 and 5 days after birth. By the end of first week of life and throughout the first month of life, chlamydia becomes the most frequent cause of conjunctivitis.

139. cRosen 5th, Chapter 66

Your next step should be a fluorescein staining of the reddened eye, followed by examination with a cobalt blue light. Corneal abrasion is a common cause of inconsolable crying in an otherwise normal infant.

140. aRosen 5th, Chapter 165, p. 2297-2301

The infant with a history of nonbilious projectile vomiting must be considered to have pyloric stenosis. Physical examination usually demonstrates a hungry infant who has failed to gain weight over the past several weeks or has lost weight. Surgery is the treatment of choice although there are reports of success with IV and oral atropine, a

141. dRosen 5th, Chapter 164, p.2283-2287

Tetralogy of Fallot is one example of cyanotic heart disease associated with decreased pulmonary blood flow. Transposition of the great vessels exemplifies cyanotic heart disease with increased pulmonary blood flow. Other congenital cardiac lesions causing cyanosis are truncus arteriosus; tricuspid atresia; and total anomalous venous return. They are known collectively as the “terrible T’s.”

142. eRosen 5th, Chapter 67, p. 931-932

A thorough and atraumatic cleansing of the ear canal is the most important part of therapy. For mild infections, dry mopping using a small tuft of cotton attached to a wire applicator is sufficient and may be curative. The tympanic membrane of a child with otitis externa may be as red and distorted as that of a child with otitis media, although mobility of the tympanic membrane is normal or only slightly decreased in otitis externa. Visualization of the tympanic membrane may be difficult because of edema of the external auditory canal.

52

Page 17: Emergency Medicine Text Review - BluWiki - 2017 Bluwikibluwiki.com/images/5/50/Questions_100-200.doc · Web viewMetabolic acidosis without an anion gap is caused by: cyanide poisoning.

143. Which statement is true regarding Group A beta-hemolytic streptococcal (GABHS) pharyngitis:a. Experienced clinicians can accurately exclude GABHS pharyngitis with the history and physical

exam alone.b. Overuse of penicillin has resulted in emergence of resistant strains of GABHS requiring a change

in recommended first line antibiotics.c. It is inappropriate to treat sore throat with antibiotics unless there is culture-proven GABHS.d. Throat culture for GABHS has very few (less than 1%) false-negatives.e. Treatment of streptococcal pharyngitis significantly shortens the course of the disease.

144. You are evaluating a 6 week-old infant for “irritability” as reported by her mother. You find an alert but cranky infant with a rectal temperature of 95oF. The left eardrum is a little red. The baby is fussy when held, but relatively quiet when left on the bed. Which statement is correct?

a. This infant needs lumbar puncture.b. Since you found an otitis media, you have an explanation for the infant’s illness and an LP is not

necessary.c. Given a reliable mother, antibiotic therapy and close outpatient follow-up is appropriated. It is almost impossible for an infant this young to have an otitis media.e. If the fontanelles are flat, you can reliably rule out meningitis.

145. In a child with a chest x-ray that shows pneumonia, the most common physical finding would be:a. rales.b. rhonchi.c. tachypnea.d. asymmetric breath sounds.e. wheezes.

146. A 4-year-old girl presents with six days of fever. You find erythema of her hands and feet, conjunctival injection, cervical adenopathy, “strawberry” tongue, dry lips, red throat, and a morbilliform rash. You should treat this girl with:

a. penicillin.b. aspirin.c. ceftriaxone.d. amoxicillin.e. dexamethasone.

147. A 3-year-old girl has a plastic bead stuck in her ear. She kicks and screams when you try to remove the bead and, even after sedation, you are unsuccessful in your attempts to remove the foreign body. Not wanting to cause injury, you give up and call the ENT surgeon, who tells you not to worry about it; send the patient home and have Mom call the next day for an appointment. You should:

a. follow the consultant’s instructions.b. insist that the consultant come in and see the patient.c. call a different consultant.d. sedate the patient even more deeply to allow successful foreign body removal.e. turn the child upside-down and shake her vigorously.

148. Which electrical injury is correctly paired with its resultant complication?a. low-voltage alternating current (AC) ventricular fibrillationb. lightning ventricular fibrillationc. high-voltage AC superficial burnsd. lightning compartment syndromee. high-voltage AC tetanic contraction

53

Page 18: Emergency Medicine Text Review - BluWiki - 2017 Bluwikibluwiki.com/images/5/50/Questions_100-200.doc · Web viewMetabolic acidosis without an anion gap is caused by: cyanide poisoning.

143. eRosen 5th, Chapter 70, p. 971-972

Research has clearly demonstrated the beneficial effects of early antibiotic therapy on reduction of signs and symptoms of GABHS pharyngitis. Diagnostic accuracy on the basis of clinical findings alone is reported at about 50 to 75% for children thought to have GABHS and 75 to 85% for children thought not to have GABHS. There is general agreement that clinical diagnosis alone would result in an unacceptably high rate of misdiagnosis. GABHS is highly sensitive to penicillin, and there has been no evidence of development of resistance in vitro despite decades of use. The rate of false-negative results from single throat culture is about 10%. Because clinical judgment is insufficient and rapid diagnostic tests are not always accurate and diagnose only GABHS, this disease process is often treated empirically.

144. aRosen 5th, Chapter 168, p. 2344-2350

The symptoms and signs suggestive of bacteremia and serious bacterial illness most frequently produce an overall ill appearance. Parents may note poor feeding, decreased responsiveness, or irritability in response to attempts to console. Physical examination findings suggestive of an ill appearance include poor eye contact and muscle tone, including weak suck, poor head control, and indifferent response to stimuli. The most important historical and physical findings are related to overall appearance. Bulging fontanelle, a hallmark of an increased intracranial pressure, is present in approximately 15% of neonatal acute bacterial meningitis.

145. cRosen 5th, Chapter 163, p. 2268-2270

The best physical examination finding for ruling out pneumonia in an infant or child is the absence of tachypnea. However, tachypnea is a nonspecific symptom and may occur secondary to fever, anxiety, metabolic disease, cardiac disease, or other respiratory problems. Fever can increase an infant’s respiratory rate by 10 breaths per minute for each degree centigrade of elevation. Auscultation of the lungs may reveal localized rales, wheezing, and decreased air entry in the affected area. However, auscultatory findings may not be reliable in children. In younger children, decreased breath sounds, rather than rales, are often heard, since the involved areas tend to be ventilated poorly.

146. bRosen 5th, Chapter 161 p. 2238-2239

Kawasaki syndrome is a vasculitis that causes a prolonged fever (more than 5 days), usually exceeding 39.5-40°C (104° F). Clinical findings include conjunctival injection without exudate; red, peeling lips; strawberry tongue; and pharyngitis. Early there may be painful swelling of the hands and feet; peeling of the skin of the fingers occurs late in the course. Cervical adenopathy in excess of 5 cm is common. Treatment is directed toward the amelioration of symptoms and the prevention of coronary aneurysms. Gamma-globulin 2 gm/kg intravenously should be administered over 12 hours, followed by high-dose aspirin therapy (100 mg/kg/24 hr PO given in divided doses every 6 hours for 14 days).

147. aRosen 5th, Chapter 53 p.754-756

If routine methods are unsuccessful or if the patient, is uncooperative or in too much distress, the emergency physician should cease removal efforts and refer the patient to an otolaryngologist. Primary operative intervention is frequently indicated in very young children and in those in whom the EP and the ENT specialist believe nonsurgical attempts unlikely to be successful. Inappropriately prolonged efforts at foreign object removal can result in wasted time, unnecessary patient discomfort, and high potential for complications.

148. aRosen 5th, Chapter 136, p. 2010-2014

Low-voltage alternating current generally produces ventricular fibrillation. High-voltage AC that causes greater current intensity is more likely to produce asystole. Direct current is also more likely to cause asystole. The most common arrhythmia encountered in victims who sustain cardiac arrest from electrical injury is ventricular fibrillation. The immediate cause of death due to a lightning strike is usually direct current depolarization of the myocardium and sustained cardiac asystole.

54

Page 19: Emergency Medicine Text Review - BluWiki - 2017 Bluwikibluwiki.com/images/5/50/Questions_100-200.doc · Web viewMetabolic acidosis without an anion gap is caused by: cyanide poisoning.

149. A 4-year-old non-immunized child presents with sudden sore throat, stridor, drooling, and fever. He looks sick and is sitting forward on the litter, but is alert. Pulse oximetry is 99% saturation on room air. Your next step is to:

a. attempt to visualize his epiglottis.b. send him to x-ray for lateral neck radiographs.c. immediately begin bag-valve-mask ventilation.d. immediately intubate the child nasotracheally.e. none of the above.

150. A 3-year-old boy is brought to the emergency department after choking on a watch battery. X-rays show that the battery is in his stomach. Appropriate management involves:

a. parental reassurance and home observation.b. ipecac to induce expulsion of the battery.c. cathartics to speed transit through the gastrointestinal tract.d. admission to the hospital for observation.e. laparotomy if the battery is still in the small bowel at 48 hours.

151. Which of the following fractures is most suggestive of child abuse?a. 19-month-old with a supracondylar fracture after falling off a chair.b. 22-month-old with a spiral femur fracture from a fall while running.c. 10-month-old with linear skull fracture from a fall down stairs in an infant walker.d. 4-month-old with a transverse fracture of the humerus from a fall out of bed.e. 3-year-old with a tuft fracture of the distal phalanx from the finger being closed in a door.

152. A 10-year-old boy complains of mild intermittent abdominal pain and a rash. His parents have not noted any fever. Physical examination shows a well-appearing boy with a raised purpuric rash, most prominent on the legs. His abdominal exam is normal. Temperature = 99.8oF. The other vital signs are normal. What is the most appropriate next step in the management of this patient?

a. Obtain blood for culture and start antibiotics.b. Obtain abdominal radiographs.c. Obtain a urinalysis, BUN, and creatinine.d. Perform a skin biopsy.e. Order liver function tests.

153. Concerning cold exposure injuries:a. chilblains (pernio) is a painless macular rash.b. in frostbite, the “zone of stasis” is most severely injured.c. early surgical intervention is recommended for severe frostbite.d. a body part previously affected by cold injury is immune from reinjury.e. trench foot may result in irreversible damage.

154. The most common finding in a patient with a brown recluse spider bite is:a. vomiting.b. severe muscle cramps.c. anaphylaxis.d. local tissue necrosis.e. respiratory failure.

55

Page 20: Emergency Medicine Text Review - BluWiki - 2017 Bluwikibluwiki.com/images/5/50/Questions_100-200.doc · Web viewMetabolic acidosis without an anion gap is caused by: cyanide poisoning.

149. eRosen 5th, Chapter 162

The ideal approach is to take any patient with suspected epiglottitis to the operating room, administer anesthesia, and examine the airway with a laryngoscope while the patient is anesthetized. If the diagnosis of epiglottitis is made, the patient can be intubated. If it is ruled out, the patient can be returned to the ward or the emergency department to continue the workup, secure in the knowledge that epiglottitis is not present.

150. aRosen 5th, Chapter 147, p. 2118

Button batteries that have passed the esophagus need not be retrieved in the asymptomatic patient unless the cell is not passing through the pylorus after 48 hours of observation, in which case endoscopic retrieval is the preferred option. Ipecac has no place in the management of button battery ingestion.

151. dRosen 5th, Chapter 169, p. 2379-2381

Fractures suggestive of child abuse include spiral fractures caused by torsion (twisting) of a long bone, and metaphyseal chip fractures, especially when present in infants less than 6 months of age. While this is the most suspicious of the fractures described, it must be emphasized that any of these fractures could result from abuse. Abuse must always be considered as a possibility with injuries involving young children.

152. cRosen 5th, Chapter 167

Henoch-Schönlein Purpura is an immunoglobulin A mediated systemic vasculitis involving the small blood vessels supplying the skin, gastrointestinal tract, and joints. The hallmark is a palpable, purpuric, or petechial rash most prominent on the lower extremities and extending to the buttocks, which is the presenting complaint in ~50% of patients. GI complaints are present in ~65% of patients, especially periumbilical, dull pain resulting from bleeding into the intestinal wall. 25% - 50% of children develop a self-limiting glomerulonephritis manifested by hematuria. There are no specific tests to confirm HSP, and the diagnosis can be difficult if the classic rash is absent. Screening tests such as urinalysis, BUN and creatinine, CBC, and coagulation studies may be needed to rule out other pathologic diseases. A helpful mnemonic is ARENA:A = Abdominal painR = RashE = EdemaN = NephritisA = Arthralgias / arthritis

153. eRosen 5th, Chapter 133, p. 1974

Trench foot develops slowly over hours to days and is initially reversible but if allowed to progress will become irreversible. Chilblains, or pernio, is characterized by mild but uncomfortable inflammatory lesions of the skin of bared extremities caused by chronic intermittent exposure to damp, nonfreezing ambient temperatures. The zone of stasis is the middle ground and is characterized by severe, but possibly reversible, cell damage. It is here that treatment is directed. Early surgical intervention is not indicated in the management of frostbite. Premature surgery has been an important contributor to unnecessary tissue loss and poor results in the past. Refreezing previously frozen tissue worsens the prognosis.

154. dRosen 5th, Chapter 55 p.796

Most victims of bites by the brown recluse spider do not even know they were bitten. Pain usually develops within 3 to 4 hours, and a white area of vasoconstriction begins to surround the bite. A bleb then forms in the center of this area, and an erythematous ring arises on the periphery. The lesion at this stage resembles a bull’s-eye. The bleb darkens, necroses over the next several hours to days, and continues to spread slowly and gravitationally, involving skin and subcutaneous fat.

56

Page 21: Emergency Medicine Text Review - BluWiki - 2017 Bluwikibluwiki.com/images/5/50/Questions_100-200.doc · Web viewMetabolic acidosis without an anion gap is caused by: cyanide poisoning.

155. A 25-year-old man was playing catch with his pet rattlesnake. He now complains of pain and swelling in his hand and forearm, with perioral numbness, and vomiting. His blood pressure is 90/60 mmHg. He will require:

a. fluid resuscitation with fresh frozen plasma.b. administration of 2 to 4 vials of antivenin.c. measurement of coagulation factors and platelets.d. immediate forearm fasciotomy.e. non-narcotic pain medication, to avoid masking of respiratory symptoms.

156. Acetazolamide acts by inhibiting the enzyme carbonic anhydrase. This in turn:a. reduces reabsorption of bicarbonate in the kidney, leading to bicarbonate diuresis and metabolic

acidosis, which produces compensatory hyperventilation.b. increases the blood carbon dioxide level, allowing a respiratory acidosis and compensatory

diuresis.c. slows the sodium-potassium ATPase pump, causing a natriuretic compensation for the leftward-

shifting oxygen dissociation curve.d. causes the kidney to reabsorb more bicarbonate ions, leading to a metabolic alkalosis, which

shifts the oxygen dissociation curve to the right to compensate for lower atmospheric pressures.e. prevents acute mountain sickness by an unknown mechanism.

157. A 55-year-old male diver complains of back pain and urinary retention which started about one hour after ascent from a dive. He most likely has:

a. bladder barotrauma.b. lumbar strain.c. envenomation by coelenterate species.d. nitrogen narcosis.e. decompression sickness.

158. Concerning High Altitude Pulmonary Edema (HAPE):a. it is the most lethal of the altitude illnesses. b. women are more susceptible than men.c. salt deprivation is a risk factor; use of a sleeping medication is somewhat protective. d. early symptoms include a moist cough with blood-tinged sputum; rales are universally present.e. the condition typically improves at night.

159. Choose the correct statement concerning electrical burns.a. When tissue carbonization occurs, resistance to current flow decreases even further.b. Contact with low voltage long distance communications lines and telephone can cause death in

certain circumstances.c. A very narrow range exists between the threshold of perception of current and the level above

which a person becomes unable to release the current source because of muscular tetany.d. The foot is the most common body part involved in alternating current injuries.e. Sweating increases resistance against electricity.

160. Which of these patients requires admission to a burn-care facility?a. A 35-year-old man with extensive partial-thickness burns on the back, shoulders, and buttocks.b. A 60-year-old diabetic with a full-thickness burn of the entire forearm.c. A 25-year-old woman with full-thickness burns of both hands and lower arms.d. A 40-year-old house-fire victim with multiple, small partial-thickness burns and wheezing.e. All of the above

57

Page 22: Emergency Medicine Text Review - BluWiki - 2017 Bluwikibluwiki.com/images/5/50/Questions_100-200.doc · Web viewMetabolic acidosis without an anion gap is caused by: cyanide poisoning.

155. cRosen 5th, Chapter 55,

Patients admitted to the hospital after poisonous snakebite should have serial determinations of platelets, prothrombin time, and urinalysis to check for myoglobin and hemoglobin. Daily comprehensive laboratory tests should be performed. Fasciotomy is not usually indicated unless compartment pressures are elevated. Antivenin (Crotalidae) polyvalent is the mainstay of therapy for poisonous snakebite; usually 10 vials or more are required. Debridement should probably not be performed earlier than 3 days after the bite, until the coagulopathy has resolved.

156. aRosen 5th, Chapter 138, p. 2041

Acetazolamide is a carbonic anhydrase inhibitor that induces a renal bicarbonate diuresis causing a metabolic acidosis, thereby increasing ventilation and arterial oxygenation. The diuretic effects may be of benefit for the fluid retention common in AMS. The drug also lowers CSF volume and pressure, which may play an additional role in its therapeutic and prophylactic use.

157. eRosen 5th, Chapter 137, p. 2024

The central nervous system is particularly susceptible to decompression illness because of its high lipid content. The spinal cord, especially the upper lumbar area, is more often involved than cerebral tissue. Symptoms of spinal DCS include limb weakness or paralysis, paresthesias, numbness, and low back and abdominal pain. Bladder symptoms, such as urinary retention, can occur, as well as fecal incontinence and priapism.

158. aRosen 5th, Chapter 138, p. 2042

High Altitude Pulmonary Edema (HAPE) is the most common fatal manifestation of severe high-altitude illness. It can occur, and even be fatal, at altitudes as low as 8000 feet. Many patients have a single episode of HAPE and subsequently are able to return to high altitude without a recurrence. Others who have had uneventful high-altitude exposures may have HAPE develop on a future ascent.

159. cRosen 5th, Chapter 136, p.

A very narrow range exists between the threshold of perception of current (0.2 to 0.4 mA) and the let-go current (6 to 9 mA), the level above which a person becomes unable to release the current source because of muscular tetany. When tissue carbonization occurs, resistance to current flow increases. No deaths are recorded from contact with low voltages associated with long distance communications lines (24V) or telephone lines (65V). The hand is the most common site of contact via a tool that is in contact with an AC electric source. Sweating can reduce the skin’s resistance from 10 – 40,000 ohms to 2500-300 ohms.

160. eRosen 5th, Chapter 56, p. 812, Box 56-2

1. Partial- or full-thickness burns involving >10% of body surface area (BSA) in patients under 10 or over 50 years of age.

2. Partial- or full-thickness burns of >20% of BSA in other age groups.3. Partial- or full-thickness burns with the threat of functional or cosmetic impairment

that involve face, hands, feet, genitalia, perineum, or major joints.4. Full-thickness burns of >5% of BSA in any age group.5. Electrical burns, including lightning injury.6. Chemical burns with the threat of functional or cosmetic importance.7. Inhalation injury with burns.8. Circumferential burns of the extremities or chest.9. Burn injury in patients with preexisting medical disorders that could complicate

management, prolong recovery, or affect mortality.10. Any burn patient with concomitant trauma, such as fracture.

58

Page 23: Emergency Medicine Text Review - BluWiki - 2017 Bluwikibluwiki.com/images/5/50/Questions_100-200.doc · Web viewMetabolic acidosis without an anion gap is caused by: cyanide poisoning.

161. Concerning victims of near-drowning:a. “dry drowning” without aspiration results from laryngospasm and glottal closure.b. electrolyte abnormalities contribute significantly to mortality.c. hemolysis resulting in anemia is found in more than half of fresh-water victims.d. disseminated intravascular coagulation is a common finding.e. postural drainage or the abdominal thrust (Heimlich maneuver) is recommended by most experts

as a way to remove water from the lungs and improve oxygenation.

162. After radiation exposure, the best predictor of hematopoietic involvement is the:a. Absolute neutrophil count at 24 hours. b. Absolute neutrophil count at 48 hours.c. Absolute lymphocyte count at 24 hours.d. Absolute lymphocyte count at 48 hours.e. Absolute platelet count at 24 hours.

163. A 35-year-old man presents complaining of headache, weakness, nausea, and vomiting after working with paint remover in an enclosed space. You know that:

a. a special antidote kit is required.b. treatment must continue longer in patients with this exposure than from other sources.c. the patient's oxygen-hemoglobin dissociation curve is shifted to the right.d. severe metabolic acidosis may be present.e. methylene blue may be required.

164. A 25-year-old photographer is found slumped over his workbench in the back of his store. Minutes earlier he had complained of headache, dizziness, and weakness. Presently the patient is comatose and apneic with a thready pulse. Cyanosis is absent. A venous blood sample drawn by paramedics in the field is noted to be very red. Laboratory results reveal a high anion gap acidosis. Blood gas determinations on arterial and venous blood samples reveal nearly identical pO2 values. Carboxyhemoglobin is 5%. Although the patient is intubated and being hyperventilated with 100% oxygen, the acidosis persists. What is the proper management?

a. administer methylene blue IVb. administer physostigmine and hydroxocobalamin (vitamin B12a)c. administer 2-PAM d. administer sodium nitrite IV, then infuse sodium thiosulfatee. transfer the patient to a facility with a hyperbaric chamber

165. In a mass casualty situation involving a large number of victims of a “dirty bomb,” the highest priority patients are those:

a. who are critically injured and contaminated.b. with external local body radiation only.c. with external total body radiation only.d. with internal contamination.e. with external contamination.

166. A 24-year-old man was bitten on the arm by his girlfriend’s cat six hours ago. He now has warmth and erythema at the site. The infecting organism is most sensitive to:

a. gentamicin.b. metronidazole.c. clindamycin.d. penicillin.e. fluconazole.

59

Page 24: Emergency Medicine Text Review - BluWiki - 2017 Bluwikibluwiki.com/images/5/50/Questions_100-200.doc · Web viewMetabolic acidosis without an anion gap is caused by: cyanide poisoning.

161. aRosen 5th, Chapter 139, p. 2051

In a small number of submersion victims, severe laryngospasm causes hypoxia, convulsions, and death in the absence of active aspiration (“dry” drowning). Most drowning victims do not aspirate enough fluid to cause life-threatening changes in blood volume or serum electrolyte concentrations. Although DIC has been reported, it is uncommon. Procedures to drain fluid from the lungs are ineffective and potentially dangerous because of the increased risk of vomiting and aspiration. If particulate material is obstructing the airway, the Heimlich maneuver may be of benefit but it is not indicated for the removal of fluid.

162. dRosen 5th, Chapter 140, p. 2058-2059

The absolute lymphocyte count 48 hours after exposure is a good predictor of hematopoietic involvement. If the absolute lymphocyte count is greater than 1200, it is unlikely that the patient has received a fatal dose of radiation. If the absolute lymphocyte count falls between 300 and 1200 at 48 hours, the possibility of exposure to a lethal dose of radiation should be suspected. A level in this range is an indication for hospitalization. Levels less than 300 are critical, and heroic procedures such as bone marrow transplant and the use of hematopoietic growth factors may be considered in individual cases.

163. bRosen 5th, Chapter 152, p. 2159-2162

Carbon monoxide (CO) toxicity can develop when inhaled methylene chloride vapor in paint strippers or from leaking "bubble" electric Christmas tree lights is slowly metabolized to produce CO. The elimination half-life of CO from methylene chloride is about twice that of inhaled CO because it is stored in tissues and gradually released. The binding of CO to hemoglobin transforms the oxyhemoglobin dissociation curve from a sigmoid shape to an asymptotic shape, increasing the ability of HbCO to hold on to oxygen at the remaining heme moiety sites. In CO toxicity, both the reduced oxygen carriage and the transformation of oxyhemoglobin dissociation curve impair tissue oxygen delivery. In effect, high HbCO imposes the equivalent of a sudden “chemical” anemia in the patient.

164. dRosen 5th, Chapter 153, p. 2166-2168

Cyanide in its salt form (e.g., sodium or potassium) is important in the metallurgic (e.g., jewelry) and photographic industries. Cyanide salts do not have an odor under dry conditions. Since cyanide prevents tissue extraction of oxygen from the blood, the oxygen content of venous blood approaches that of arterial blood. Clinically this may appear as the “arterialization,” or brightening, of venous blood to resemble arterial blood. Although the exact mechanism of the cyanide antidote kit is controversial, the accepted goal of therapy is to reactivate the cytochrome oxidase system by providing an alternative, high-affinity source of ferric ions (Fe3+) for cyanide to bind. Sodium nitrite and sodium thiosulfate are the active ingredients in the kit.

165. aRosen 5th, Chapter 140, p. 2060-2062

If treatment of great numbers of radiation exposed and contaminated patients is necessary, resuscitation and stabilization always takes precedence over decontamination. However, health care workers must not expose themselves. Protection of health care workers from injury takes priority over the patient's ABCs'.

166. dRosen 5th, Chapter 54, p. 775-776

Pasteurella multocida is the major pathogen found in infected cat bite wounds, isolated in up to 80%. Infection is characterized by a rapidly developing, intense inflammatory response, often within a few hours and rarely more than 24 hours after the bite. Pain and swelling are prominent. The antibiotic best used to treat the major organism in this presentation is penicillin.

60

Page 25: Emergency Medicine Text Review - BluWiki - 2017 Bluwikibluwiki.com/images/5/50/Questions_100-200.doc · Web viewMetabolic acidosis without an anion gap is caused by: cyanide poisoning.

167. Your friend has just stepped on a jellyfish while swimming. He is tearful because of the pain. You can offer temporary relief by:

a. drying the area.b. rinsing the site with ethanol.c. rinsing the site with fresh water.d. rinsing the site with milk.e. rinsing the site with salt water.

168. The vast majority of mushroom-related fatalities in North America are due to:a. coprine-induced disulfiram-like reaction when consumed with ethanol.b. cyclopeptide-induced liver and renal failure.c. monomethylhydrazine-induced seizures and hepatorenal failure.d. muscarine-induced "SLUDGE" syndrome.e. psilocybin-induced hallucinations and seizures.

169. A 16-year-old girl ingested an unknown plant an hour ago. Now she is tachycardic with mydriasis, decreased bowel sounds, altered vision, and abnormal mental status. After ensuring the basic ABCs, your next action should be:

a. intravenous atropine.b. GI decontamination.c. isotonic intravenous fluid administration.d. subcutaneous physostigmine.e. intramuscular glucagon.

170. A hiker on the Appalachian Trail is bitten on the hand by a snake, which was positively identified as having red bands next to yellow bands. She describes mild pain at the site of the bite, but is otherwise asymptomatic. Your recommended treatment is:

a. admission to the hospital for observation only.b. discharge after extended observation.c. discharge after routine wound care.d. administration of antivenin if the patient becomes symptomatic.e. immediate administration of antivenin.

171. Which is the typical sequence of cardiac arrhythmic deterioration in a patient whose core temperature is below 30oC (86oF).

a. atrial fibrillationventricular fibrillationpulseless electrical activityasystoleb. sinus bradycardiaventricular tachycardiaventricular fibrillationasystolec. sinus bradycardiaatrial fibrillationventricular fibrillationasystoled. atrial fibrillationatrial flutterventricular fibrillationasystolee. sinus bradycardiaatrial flutteratrial fibrillationfine ventricular fibrillation

172. A 25-year-old man notices burning pain in his arm while retrieving logs from a woodpile. Within six hours, he has developed a necrotic area surrounded by an erythematous ring. He then notices fever, myalgias, nausea and generalized weakness. Of those listed, the most likely cause is:

a. acute contact dermatitis.b. brown recluse spider bite.c. splinter from maple bark.d. deer tick bite.e. black widow spider bite.

61

Page 26: Emergency Medicine Text Review - BluWiki - 2017 Bluwikibluwiki.com/images/5/50/Questions_100-200.doc · Web viewMetabolic acidosis without an anion gap is caused by: cyanide poisoning.

167. eRosen 5th, Chapter 55, p. 797-798

Jellyfish envenomation sites should be washed off with salt water from the ocean, as fresh water is reported to worsen the pain.

168. bRosen 5th, Chapter 144, p. 2203-2205

Cyclopeptides such as amatoxin, found in certain amanita species, produce a three-phase pattern of toxicity with early GI effects followed by a quiescent phase during which the patient may be released from medical care. The devastating third phase results in severe hepatic and sometimes renal toxicity. Ninety-five percent of North American deaths from mushroom poison are due to these toxins. While monomethylhydrazine poisoning from the Gyromitra species may have 10 – 40% mortality, this poisoning is much less common and thus yields fewer fatalities. Poisoning due to mushrooms containing muscarine, psilocybin, or coprine generally responds well to supportive measures and antidotal therapy.

169. bRosen 5th, Chapter 144, p. 2081-2081Chapter 158, p. 2199-2201

Anticholinergic toxicity commonly occurs with jimsonweed ingestion. This plant contains belladonna alkaloids such as atropine, scopolamine, and ecgonine. After assuring the ABC's, treatment includes GI decontamination with emesis or lavage, activated charcoal, and supportive care (IV fluids, external cooling, and restraints for patient protection). GI decontamination may be useful for up to 48 hours after ingestion if the patient remains symptomatic. Isotonic fluid administration is the first line treatment for hypotension. Many authorities recommend treatment of "full-blown" anticholinergic syndrome with physostigmine, but reports of asystole, ventricular arrhythmia, hypotension, bronchospasm, and seizures have occurred.

170. eRosen 5th, Chapter 55, p. 789-792

“Red on yellow, kill a fellow; red on black, venom lack.” This applies to women victims, too. Anyone bitten by the Eastern coral snake (Micrurus fulvius) should be given the antivenin even before any symptoms develop. The toxicity of this venom has a rapid onset, and once the symptoms develop, it may be too late to reverse the effects with antivenin. The recommended dose is three to five vials in 300 to 500 ml of normal saline.

171. cRosen 5th, Chapter 134, p. 1989-1990

Patients are at risk for dysrhythmias at body temperatures below 30°C (86°F); the risk increases as body temperature decreases. Although various dysrhythmias may occur at any time, the typical sequence is a progression from sinus bradycardia to atrial fibrillation with a slow ventricular response, to ventricular fibrillation, and ultimately, to asystole.

172. bRosen 5th, Chapter 55, p. 795-796

As the name implies, the brown recluse spider is a reclusive organism. Humans encounter this spider in attics, storage sheds, crawl spaces, and woodpiles. Encounters with humans are uncommon. Brown recluse spiders are not aggressive and bite humans in self-defense. The venom results in epidermal and subcutaneous necrosis. This reaction is a local process around the bite site. About 6 to 8 h after the bite, pain associated with a red-to-violaceous discoloration develops.

62

Page 27: Emergency Medicine Text Review - BluWiki - 2017 Bluwikibluwiki.com/images/5/50/Questions_100-200.doc · Web viewMetabolic acidosis without an anion gap is caused by: cyanide poisoning.

173. When resuscitating a hypothermic patient whose core temperature is <85oF you should:a. administer prophylactic lidocaine.b. move the patient as little as possible.c. pronounce the patient dead if there is no cardiac response after rewarming to 90oF.d. use Ringer's lactate as the intravenous fluid of choice.e. massage extremities and apply heating blanket.

174. A diver was stung by a stingray and has severe pain, which you can relieve by:a. applying ice packs.b. applying vinegar.c. immersing his leg in hot water.d. urinating on his leg.e. using a topical slurry of sodium bicarbonate.

175. The delayed complications of fetal hemorrhage into maternal circulation are of most concern in a woman:

a. taking steroids.b. who is Rh-negative.c. with beta-thalassemia.d. with alpha-thalassemia.e. with sickle trait.

176. A 36-week pregnant woman presents in shock with painful vaginal bleeding and a hard, tender uterus. She most likely has:

a. eclampsia.b. endometritis.c. placenta previa.d. threatened abortion.e. abruptio placenta.

177. A 28-year-old woman is brought to your Emergency Department by ambulance in cardiopulmonary arrest after an automobile collision. Cardiopulmonary resuscitation was started approximately 4 minutes prior to arrival. Her husband is in the next room, crying hysterically. “She’s due with our first baby next month.” Judging from her fundal height you estimate that the fetus is at 34-weeks gestational age. You are the only physician in the hospital, covering a Level II trauma center. Your most appropriate next action is:

a. page the obstetrician.b. start a tocolytic agent and page the obstetrician.c. pronounce the patient dead.d. continue ACLS and page the trauma surgeon.e. perform perimortom cesarean delivery.

178. The earliest that a serum beta-hCG test can detect pregnancy is:a. shortly before a missed period.b. during the time of an expected period.c. 2-weeks after a missed period.d. 4-weeks after a missed period.e. 6-weeks after a missed period.

63

Page 28: Emergency Medicine Text Review - BluWiki - 2017 Bluwikibluwiki.com/images/5/50/Questions_100-200.doc · Web viewMetabolic acidosis without an anion gap is caused by: cyanide poisoning.

173. bRosen 5th, Chapter 134, p. 1989-1994

The hypothermic myocardium is extremely irritable, and ventricular fibrillation may be induced by a variety of manipulations and interventions that stimulate the heart, including rough handling of the patient. The activity of antiarrhythmic and cardioactive drugs is unpredictable in hypothermia, and the hypothermic heart is relatively resistant to atropine, pacing, and countershock.

174. cRosen 5th, Chapter 55, p. 797-799

The success of therapy for stings from marine animal spines depends on rapid initiation. Treatment is directed at combating the effects of the venom, alleviating pain, and preventing infection. The wound should be irrigated immediately, and any visible pieces of the spine or integumentary sheath should be removed. As soon as possible, the wound should be immersed in hot water to tolerance [45°C (113°F)] for 30 to 90 min or until there is pain relief. During the hot-water soak, the wound should be explored and foreign material removed.

175. bRosen 5th, Chapter 172, p. 2424-2425

Rh immunization occurs when an Rh-negative female is exposed to Rh-positive blood during pregnancy or delivery. The incidence of sensitization is about 16% when the Rh-negative woman and Rh-positive fetus are ABO-compatible and 1% to 2% if they are ABO-incompatible. Sensitization occurs most commonly at the time of delivery, but transplacental hemorrhage can occur during threatened miscarriage (even without fetal loss), spontaneous miscarriage, surgery for ectopic pregnancy, and amniocentesis. Anti-D immune globulin (RhoGAM) should be administered when these events occur. RhoGAM is routinely administered to Rh-negative mothers (if the father is Rh-positive or his status is unknown) at about the 28th week of gestation to protect the mother from spontaneous sensitization, which occurs during the third trimester. RhoGAM has also been used to prevent further deterioration of fetal anemia in Rh-D immunizations if started before severe fetal anemia and imminent hydrops fetalis arise. Minidose RhoGAM has been used in some patients with early miscarriage or trauma.

176. eRosen 5th, Chapter 172 p. 2420

Abruptio placentae, or separation of the placenta from the uterine wall, accounts for about 30% of episodes of bleeding during the second half of pregnancy. Vaginal bleeding occurs in 80% of patients with abruptio placentae. Blood is characteristically dark and the amount is often insignificant, although the mother may have hemodynamic evidence of significant blood loss. Uterine tenderness or pain is seen in about two thirds of women; uterine irritability or contractions are seen in one third. With significant placental separation, fetal distress occurs and the maternal coagulation cascade may be triggered causing DIC.

177. eRosen 5th, Chapter 31, p. 264

The need to perform perimortem cesarean delivery in cases of maternal cardiac arrest arises extremely infrequently. The time to delivery from the onset of maternal arrest was found to be critical to fetal survival with good neurologic outcome. Excellent outcomes were reported when delivery took place within 5 min of maternal death. Survival was unlikely if delivery occurred after 20 min of maternal arrest. Consideration of perimortem cesarean delivery must be made only after immediate and optimal advanced maternal cardiopulmonary resuscitative measures have been instituted. Successful maternal revival following fetal delivery has been reported. Improved venous return to the central circulation, increased maternal oxygen delivery following removal of the high uterine demand, and decreased pooling of blood in the uteroplacental circulation have all been suggested explanations.

178. aRosen 5th, Chapter 171, p. 2404-2405

The quantitative beta-hCG analysis may be very helpful. It is first detectable as early as 9 to 11 days following ovulation (usually 24 days after the last menstrual period) and reaches 200 IU/mL at the expected time of menses.

64

Page 29: Emergency Medicine Text Review - BluWiki - 2017 Bluwikibluwiki.com/images/5/50/Questions_100-200.doc · Web viewMetabolic acidosis without an anion gap is caused by: cyanide poisoning.

179. A 20-year-old woman is 14 weeks pregnant. She has had vaginal spotting for eight hours. Her vital signs are normal, including orthostatic readings. Her vaginal vault is normal; her cervical os is closed. Because she is obese, you cannot estimate uterine size or adnexal structures, but feel that there is no tenderness. Fetal heart tones are heard with a Doppler stethoscope at a rate of 150 beats per minute. Her hematocrit is 35%. Optimal management includes:

a. conjugated estrogen 20 mg IM and observation for eight hours.b. emergent gynecologic consultation to admit the patient for bed rest and fetal monitoring.c. transvaginal ultrasound followed by fetal monitoring for 12 hours.d. type and screen, administer Rho (D) Immune Globulin if Rh-negative.e. discharge home on bed rest with OB follow-up in 24 hours.

180. A 23-year-old G3P2 patient who is 34 weeks by dates presents with acute abdominal pain and heavy dark vaginal bleeding. Her blood pressure is 78/32 mmHg. When the nurse draws blood and starts an intravenous line, the patient oozes blood from the IV site. She also has a nosebleed. A DIC panel will probably show:

a. diminished fibrin split products.b. negative d-dimer.c. prolonged prothrombin time.d. elevated fibrinogen levels.e. normal platelets.

181. A 28-year-old female insulin-dependant diabetic who is 16 weeks pregnant complains of 2 days of vomiting, shortness of breath, and high blood sugar. She has no known allergies. Her examination shows oral temperature 100.0oF, heart rate 130 beats / minute, respiratory rate 28 / minute, and blood pressure 100/70 mmHg. Her breath smells like fingernail polish, and her urine dips strongly positive for ketones. Her bedside fingerstick glucose is 287 mg/dl. You suspect diabetic ketoacidosis, knowing that in pregnancy:

a. DKA may develop at lower glucose values. b. hyperemesis is seldom a precipitant. c. maternal insulin and counter-regulatory hormones cross the placenta. d. it is more frequently seen in patients with gestational diabetes than in those with juvenile

diabetes. e. serum pH may be deceptively low.

182. A 15-year-old female presents by ambulance, complaining of sudden abdominal pain. She is pale and diaphoretic. She is initially afebrile with heart rate 140 beats / minute, respiratory rate 20 / minute, and blood pressure 60/0 mmHg. Her lungs are clear, and her heart is tachycardic. Her abdomen is soft with bowel sounds present and no masses palpable. Her stool is brown stool and negative for blood. Pelvic exam shows blood in the vault. She has a nonfocal neurologic exam, but she loses consciousness as you examine her. Bedside glucose is 180 mg/dl. You assure the ABCs and start two large bore antecubital intravenous lines. She does not respond to 0.8 mg of naloxone. Your next step is:

a. call general surgeon, transfer to OR with probable ruptured appendix.b. call obstetrics for emergent transfer to OR, type and screen.c. send patient to radiology for abdominal and vaginal ultrasound.d. await results of serum pregnancy and quantitative beta-hCG.e. stat CT scan of abdomen and pelvis.

65

Page 30: Emergency Medicine Text Review - BluWiki - 2017 Bluwikibluwiki.com/images/5/50/Questions_100-200.doc · Web viewMetabolic acidosis without an anion gap is caused by: cyanide poisoning.

179. dRosen 5th, Chapter 171, p. 2403-2406Chapter 172, p. 2417-2419

In a woman with first trimester pregnancy and vaginal bleeding, you must rule out ectopic pregnancy and threatened miscarriage. A transvaginal ultrasound is not indicated because the presence of fetal heart tones confirms the dates. Emergent obstetric consultation is not indicated because she is hemodynamically stable. Estrogens are not appropriate in pregnancy. It is important to assess the woman's Rh status and give RhoGAM if appropriate.

180. cRosen 5th, Chapter 116, p.1698,Table 116-3

MOST USEFULProthrombin time ProlongedPlatelet count Usually lowFibrinogen level Low

HELPFULaPTT Usually prolongedThrombin clot time ProlongedFragmented RBCs Should be presentFeibrin split products ElevatedD-dimers Elevated

181. aRosen 5th, Chapter 173, p. 2440-2441

About 10% of insulin-dependent diabetics will develop ketoacidosis at some point during pregnancy. DKA occurs more rapidly and at lower glucose levels in pregnant patients as compared with nonpregnant patients. Hyperemesis and noncompliance or errors in insulin dosage are the most common precipitants. Admission is often indicated to correct dehydration and more carefully adjust glucose control. The serum pH may be deceptively normal in the pregnant patient, because the initial pH tends to be higher in pregnancy due to physiologic hyperventilation. DKA is rare in patients with gestational diabetes, and insulin and counter-regulatory hormones do not cross the placenta.

182. bRosen 5th, Chapter 172, p. 2416-2419

Unstable patients suspected of ectopic pregnancy should receive resuscitation, urgent consultation, and operative intervention. Surgery may be both diagnostic and therapeutic if an EP is found or may reveal another cause for the patient’s condition. When bedside ED sonography is available, it may be valuable even in unstable patients if it does not interfere with resuscitation, consultation, and rapid transfer to the operating room.

66

Page 31: Emergency Medicine Text Review - BluWiki - 2017 Bluwikibluwiki.com/images/5/50/Questions_100-200.doc · Web viewMetabolic acidosis without an anion gap is caused by: cyanide poisoning.

183. Concerning the diagnosis of appendicitis during pregnancy:a. diagnosis of appendicitis is delayed because x-rays are not taken.b. positive urinary WBC esterase and bacterial nitrite are usually seen in appendicitis.c. the appendix rotates from McBurney's point as pregnancy progresses, ultimately lying near the

midline in the upper abdomen.d. the incidence of appendicitis is increased in pregnancy.e. the incidence of perforated appendix is increased in pregnancy.

184. A woman who is 30-weeks pregnant is involved in a motor vehicle crash and sustains a spiral fracture of the right humerus. She is hemodynamically stable and has an otherwise normal physical exam. In addition to treating her orthopedic injury, you know she needs:

a. abdominal CT scanb. ECGc. fetal monitoringd. urine pregnancy teste. urine toxicology screen

185. A 26-year-old woman has dysuria for one week. She denies fever, nausea, vomiting, and back pain. She does admit to having a new sexual partner over the past month. Her pelvic examination shows a small amount of whitish discharge from the cervical os but is otherwise normal. Urinalysis shows 10 WBCs, 0 RBCs, no epithelial cells, and no bacteria. Her most likely diagnosis is:

a. pyelonephritis secondary to E. coli infection.b. cystitis secondary to E. coli infection.c. urethritis secondary to Chlamydia infection.d. cervicitis secondary to gonorrheal infection.e. vaginitis secondary to candida infection.

186. You are using serial quantitative beta-hCG tests to assess the health of a fetus in a patient with first trimester threatened abortion. You know that in a healthy pregnancy with a normal fetus the level:

a. doubles every two to three days.b. doubles every seven days.c. triples every two to three days.d. triples every seven days.e. will not change by more than 10% during the first 12 weeks of pregnancy.

187. You are arranging transportation for a 23-year-old woman who fell down some stairs and fractured her pelvis. She is 28 weeks pregnant by bedside ultrasound. Her blood pressure is 64 palpable. The position best for her to travel is:

a. 60o Trendelenberg.b. tilted 15o to the left.c. tilted 45o to the right.d. with her head elevated 45o.e. lying on her right side.

188. The most common manifestation of gonococcal infection in children is:a. salpingitis.b. cervicitis.c. pharyngitis.d. vaginitis.e. conjunctivitis.

67

Page 32: Emergency Medicine Text Review - BluWiki - 2017 Bluwikibluwiki.com/images/5/50/Questions_100-200.doc · Web viewMetabolic acidosis without an anion gap is caused by: cyanide poisoning.

183. eRosen 5th, Chapter 172, p. 2425-2427

Appendicitis is the most common surgical emergency in pregnancy. Appendectomy is one of the most common laparoscopic procedures performed during pregnancy, second only to cholecystectomy. The incidence of appendicitis in pregnancy is unchanged from the nonpregnant state, but delays in diagnosis contribute to an increased rate of perforation. During the first half of pregnancy, diagnostic findings are usually similar to those in the nonpregnant female, but the clinical picture becomes less classic during the second half. During pregnancy the appendix is displaced counterclockwise out of the right lower quadrant so that by the last trimester, the appendix is located deep in the right upper quadrant, superior to the iliac crest. Proximity to the ureter, and during later pregnancy to the kidney itself, leads to an increased incidence of sterile pyuria in the pregnant patient who has appendicitis, further confounding the diagnosis.

184. cRosen 5th, Chapter 31, p. 262-264

Fetal evaluation in the secondary survey focuses on the fetal heart rate and noting any fetal movement. Once the presence of fetal heart tones has been confirmed, intermittent monitoring of fetal heart rate is sufficient for the pre-viable fetus. If the fetus is viable (i.e., 24 weeks or more), continuous external monitoring should be initiated quickly and maintained throughout all diagnostic and therapeutic procedures. Such monitoring can also benefit the mother because fetal hemodynamics are more sensitive to decreases in maternal blood flow and oxygenation than are most maternal measures. Fetal distress can be a sign of occult maternal distress. Signs of fetal distress include an abnormal baseline rate, decreased variability of heart rate, and fetal decelerations after contractions.

185. cRosen 5th, Chapter 94, p. 1395-1396

Chlamydial urethritis may be present in up to 20% of women with dysuria. The history of a new sexual partner, the gradual onset of symptoms, the discharge from the os, and the sterile pyuria are all consistent with chlamydial urethritis. The lack of fever, vomiting, and back pain make pyelonephritis less likely.

186. aRosen 5th, Chapter 172, p. 2418

Serum beta-HCG levels normally double every 1.8 to 3 days for the first 6 to 7 weeks of pregnancy, beginning 8 to 9 days after ovulation.

187. bRosen 5th, Chapter 31, p. 256 & 264

For pregnant patients beyond 20 weeks of gestation who must be transported in the supine position or in whom spinal immobilization is indicated, a wedge should be placed under the right hip area tilting the patient toward her left side to avoid hypotension from inferior vena cava compression by the gravid uterus.

188. dRosen 5th, Chapter 93, p. 1395

Vaginitis is the most common gonococcal infection in children. The thin, friable genital mucosa of the prepubescent girl is susceptible to gonococcal infection, resulting in copious, purulent vaginal discharge. Obtain specimens from the child at the vaginal introitus. Vaginitis, scalp infection, bacteremia, arthritis, meningitis, and endocarditis are manifestations of gonorrhea in the newborn.

68

Page 33: Emergency Medicine Text Review - BluWiki - 2017 Bluwikibluwiki.com/images/5/50/Questions_100-200.doc · Web viewMetabolic acidosis without an anion gap is caused by: cyanide poisoning.

189. HELLP stands for:a. hepatitis – elevated lipids – low plateletsb. hemolysis – elevated lipids – low plateletsc. hepatitis – elevated liver enzymes – low plateletsd. hemolysis – elevated liver enzymes – low plateletse. herpes – extrophic labia – lumbar pernio

190. In a patient with genital herpes:a. HSV-1 accounts for up to 50% of the cases of genital herpes.b. systemic acyclovir decreases the frequency of recurrences.c. systemic symptoms are common with the initial presentation of genital herpes.d. Tzanck smears are positive in the majority of cases.e. cultures of fluid obtained from herpes vesicles are positive only a third of the time.

191. In the patient with sinusitis:a. CT scans of the sinuses are the "gold standard" for diagnosing the disease.b. ethmoid sinusitis pain is made worse when the patient sits up. c. maxillary sinusitis pain is made worse by placing the patient supine.d. phenylephrine or oxymetazoline decongestants should be avoided, as they can irritate the nasal

mucosa.e. the Water's view is most helpful in finding ethmoid sinusitis.

192. The most common agent causing infectious pharyngitis in an adult is:a. Chlamydia trachomatis.b. group A beta-hemolytic streptococcus.c. Neisseria gonorrhoeae.d. viral.e. pneumococcus.

193. A 65-year-old woman has swelling at the base of her tongue and a "bull neck". She is an insulin-dependent diabetic. You find brawny induration of the floor of the mouth. She is ill appearing but is not in any respiratory distress. You should:

a. incise and drain the area of swelling.b. admit the patient to the ICU for close monitoring of possible airway obstruction.c. start IV antibiotics in the ED and admit her to the general medical floor.d. discharge home on outpatient antibiotics with ENT follow-up the next daye. consult a dentist for probable tooth extraction.

194. Ménière’s disease is characterized by vertigo and:a. fever.b. hearing loss.c. headache.d. blurred vision.e. facial nerve paralysis.

195. Concerning nosebleeds:a. anterior epistaxis accounts for 75% of nosebleeds.b. anterior epistaxis requires immediate consultation with an otolaryngologist.c. anterior epistaxis usually originates from the turbinates of the lateral wall of the nasal cavity.d. posterior epistaxis is less likely to require hospitalization than anterior epistaxis.e. posterior epistaxis usually occurs from the posterior branch of the sphenopalatine artery.

69

Page 34: Emergency Medicine Text Review - BluWiki - 2017 Bluwikibluwiki.com/images/5/50/Questions_100-200.doc · Web viewMetabolic acidosis without an anion gap is caused by: cyanide poisoning.

189. dRosen 5th, Chapter 172, p. 2422-2423

The HELLP syndrome (an acronym for hemolysis, elevated liver enzymes, and low platelets) is an important clinical variant of preeclampsia that has a predilection for the multigravid patient, in contrast to the primigravida, in whom preeclampsia is more common. In the HELLP syndrome, the blood pressure is variable and may not be elevated initially. This fact, combined with the usual complaint of epigastric or right upper quadrant pain, makes it easy to mistake the HELLP syndrome for other causes of abdominal pain, such as gastroenteritis, hepatitis, pancreatitis, cholecystitis, or pyelonephritis.

190. cRosen 5th, Chapter 93, p. 1392

Overall, 85 to 90% of genital herpes infections are caused by HSV-2. There may be both local and systemic manifestations. Cultures are positive 85 to 95% of the time. Usually, initial infection is more severe and lasts longer than do subsequent recurrences. A Tzanck smear stained with either Wright or Giemsa stain is positive in up to 50% of cases. Systemic antiviral agents provide partial control of the signs and symptoms and accelerate healing of the lesions, but do not affect the frequency or severity of recurrences.

191. aRosen 5th, Chapter 70, p. 981-984

The best method for imaging the sinuses is to perform a CT. The Waters view is a plain x-ray that best shows the maxillary sinuses and can provide good views of the frontal sinuses. Maxillary sinusitis is usually made worse when the patient leans forward, while assuming the supine position exacerbates ethmoid sinusitis. Decongestants are an important part of treatment, allowing drainage of the sinuses. Prolonged use of these agents can cause a rebound inflammation on cessation.

192. dRosen 5th, Chapter 70, p. 969-971.

Causal agents of pharyngitis include viruses, bacteria, fungi, and parasites. Most often, viruses are the culprits. Rhinovirus and adenovirus are the most common, but Epstein- Barr virus, herpes simplex virus, influenzavirus, parainfluenzavirus, and coronavirus are responsible for about 5% of the infections.

193. bRosen 5th, Chapter 65, 895-896Chapter 70, p. 977-978

Cellulitis of bilateral submandibular spaces and the lingual space is called Ludwig´s angina and is potentially life threatening. It is a rapidly spreading cellulitis that results in brawny induration of the suprahyoid region and elevation of the tongue. Involvement of the floor of the mouth pushes the tongue posteriorly. Epiglottic involvement is not uncommon. As a result, airway compromise is the immediate primary concern. The primary focus of initial management is maintenance of a patent airway. Timely administration of high- dose penicillin and metronidazole or cefoxitin is essential. Immediate oral and maxillofacial surgical consultation and hospitalization for incision and drainage and intubation as indicated are necessary.

194. bRosen 5th, Chapter 13, p. 125-127

With Ménière’s disease the onset of vertigo is usually sudden, and duration ranges from 20 minutes to 12 hours. It is associated with nausea, vomiting, and diaphoresis. The frequency of attacks can vary from several times per week to several times per month. Between attacks, the patient is usually well, although deafness may persist. Other hallmarks of the diagnosis include associated symptoms such as roaring tinnitus, diminished hearing, and fullness in one ear.

195. eRosen 5th, Chapter 67, p. 933-935

Anterior epistaxis is responsible for 90% of nosebleeds. It usually arises from the anterior-inferior nasal septum from vessels known as Kiesselbach's plexus. Although posterior epistaxis requires consultation with an otolaryngologist and hospitalization, most anterior bleeds can be managed on an outpatient basis without immediate consultation of a specialist. The most common site of posterior bleeds is the posterior branch of the sphenopalatine artery.

70

Page 35: Emergency Medicine Text Review - BluWiki - 2017 Bluwikibluwiki.com/images/5/50/Questions_100-200.doc · Web viewMetabolic acidosis without an anion gap is caused by: cyanide poisoning.

196. Necrotizing external otitis is almost always caused by:a. group A beta-hemolytic streptococcus.b. Staphylococcus aureus.c. E. coli.d. Pseudomonas aeruginosa.e. aspergillus.

197. Central retinal vein occlusion:a. causes brief, transient blindness or flickering vision.b. causes a pale retina and optic disc with “boxcar” segmentation of the retinal veins.c. is secondary to embolic phenomenon.d. is treated with eye massage, acetazolamide, timolol, and increasing the inhaled pCO2.e. has a wide range of clinical appearances.

198. A 4-year-old child has purulent nasal discharge from the left nostril for 4 days. She flails her arms and shakes her head vigorously when you attempt to examine her, but you notice a bead from a toy necklace partially occluding the left nostril. You should:

a. send her home on oral antibiotics with assurance to mom that the object will dislodge itself.b. instill topical vasoconstrictor into the nostril and attempt blind removal with a right angle hook.c. obtain an X-ray of the nasal bones.d. under conscious sedation, grasp the object with a suction catheter taking precautions against

pushing the foreign body further back into the nasopharynx.e. consult ENT for referral to the operating room.

199. An 18-year-old man presents with right ear pain and discharge. Temperature 39oC (102.2oF). You find a perforated right tympanic membrane with purulent drainage, and a tender swelling over the mastoid area. The most appropriate treatment is:

a. admit, culture drainage, start IV antibiotics, obtain urgent otolaryngology consultation.b. admit patient on antibiotic ear drops with routine consult otolaryngology.c. discharge on oral antibiotics to follow-up with an otolaryngologist in one week.d. incise and drain mastoid area, then discharge on oral antibiotics.e. topical sulfacetamide drops for 7 days.

200. A 32-year-old woman is struck in the left eye with a tennis ball. She complains of pain but denies flashes of light, floaters, diplopia, or a decrease in vision. On initial evaluation her visual acuity is 20/20 vision in each eye, she has orbital emphysema, and her extraocular muscles and globe are intact. Shortly after arrival she complains of suddenly decreased visual acuity. You should now:

a. ballotte the globe in an attempt to dislodge the clot causing the central retinal artery occlusion.b. intraorbital needle aspiration or lateral canthotomy with cantholysis to release pressure under the

orbit. c. ophthalmologic consult for traumatic retinal tear with vitreous hemorrhage.d. topical cycloplegics (5% homatropine) to the affected eye for treatment of traumatic iridocyclitis

with an ophthalmologic follow-up.e. anterior chamber paracentesis.

71

Page 36: Emergency Medicine Text Review - BluWiki - 2017 Bluwikibluwiki.com/images/5/50/Questions_100-200.doc · Web viewMetabolic acidosis without an anion gap is caused by: cyanide poisoning.

196. dRosen 5th, Chapter 67, p. 932

Previously known as malignant otitis externa because of its high mortality, necrotizing external otitis is an extremely aggressive form of otitis externa. It occurs primarily in adults with diabetes mellitus but has also been seen rarely in immunocompromised children. Pseudomonas is the predominant pathogen, but S. aureus, S. epidermidis, Proteus mirabilis, Klebsiella, Aspergillus, and Salmonella have all been described.

197. eRosen 5th, Chapter 66, p. 921

Central retinal vein occlusion (CRVO) symptoms are similar to those of central retinal artery occlusion (CRAO) in that there is a painless loss of vision. Brief transient blindness is uncommon. A pale retina and optic disc with boxcar segmentation are seen in CRAO. The degree of vision loss depends on the degree of ischemia and ranges from mild to severe. CRAO results from embolic phenomenon, but not CRVO. No treatment is necessary acutely for CRVO other than watching for elevation of intraocular pressure consistent with neovascular glaucoma. Eye massage, acetazolamide, timolol, and increasing the PCO2 are all indicated treatments for CRAO.

198. dRosen 5th, Chapter 53, p. 756-757

With conscious sedation, a majority of nasal foreign bodies can be removed. If attempts are unsuccessful, the patient should follow-up with an otolaryngologist within one to two days.

199. aRosen 5th, Chapter 67, p. 932-933

Mastoiditis requires admission for antibiotic therapy. Antibiotic choices include semisynthetic penicillin combined with chloramphenicol, or a third-generation cephalosporin such as cefuroxime (50 to 150 mg/kg/day), or ceftriaxone (50 to 75 mg/kg/day), usually for 1 week. Surgical procedures may range from myringotomy drainage and tympanostomy tube placement to mastoidectomy and drainage for more extensive disease progression. Mastoidectomy is required in approximately half of mastoiditis cases.

200. bRosen 5th, Chapter 66, p. 909

Orbital emphysema associated with orbital fractures is usually a benign, self-limited condition. This patient however, is complaining of a sudden decrease in visual acuity in the traumatized eye. The emergency physician must consider that air may have built up under pressure in the orbit, causing cessation of blood flow in the central retinal artery. The air must be released immediately or the patient may lose her vision. This is done by performing a lateral canthotomy with cantholysis or intraorbital needle aspiration of the trapped air. Ballottement of the globe will not benefit this patient because the etiology of her visual loss is not embolic occlusion of the retinal artery. Iridocyclitis following trauma is painful but should not be associated with loss of visual acuity.

72


Recommended